• Now Trending:
  • Several students mentore...
  • Kenyan Government to imp...
  • TSC to continue CBC reto...
  • How do download the 2024...

Biology KCSE Essay Questions and Answers Paper 2; Over 1,000

KCSE BIOLOGY PAPPER TWO 231/2 -ESSAYS FROM 1995 -2023

  a). Describe how insect pollinated flowers are adapted to pollination ( KCSE1995)

  • Progesterone
  • Luteinizinghormone
  • Describe how excretion takes place in: ( KCSE1995)
  • MammalianKidneys
  • Greenplants
  • a).Explainhowthemammalianskinisadaptedtoperformitsfunctions (20marks; KCSE 1996)

b). Describe how new plants arise by asexual reproduction (20 marks; KCSE 1996)

  • a). What is parasitism? (KCSE1997)

b). Describe how the tapeworm is adapted to a parasitic mode of life (KCSE 1997)

  • a). What is meant by the term digestion? (KCSE1997 )

b). Describe how the mammalian small intestine is adapted to its function (KCSE 1997)

  • Discuss the various evidences, which show that evolution has taken place (20 marks; KCSE 1998)
  • Explain how the mammalian intestines are adapted to perform their function (20 marks; KCSE 1998)
  • a). Describethe:
  • Process of inhalation in mammals ( KCSE 1999)
  • Mechanisms of opening and closing of stomata in plants (KCSE1999)

b). Explain how the various activities of man have caused pollution of air (20 marks; KCSE 1999)

  • a). Describe the role of hormones in the human menstrual cycle (20 marks; KCSE2000)

b). How are leaves of mesophytes suited to their functions (20 marks; KCSE 2000)

  • a). State the functions of the following parts of the mammalian ear; (KCSE2001)
  • Tympanicmembrane
  • Eustachiantube
  • Earossicles

b). Describe how semicircular canals perform their functions (KCSE 2001 )

  • a). Describe the process of fertilization in a flowering plant ( KCSE2001)

b). State the change that take place in a flower after fertilization ( KCSE 2001)

  • a). Describe the role of hormones in the growth and development of plants (20 marks; KCSE 2002)
  • a). Name three types of skeletons found in multicellular animals ( KCSE2002)

b). Describe how the cervical, lumbar and sacral vertebrae are suited to their functions

Also read..

BIOLOGY FORM ONE SUMMARIZED NOTES

Biology Form one to four exams, notes and revision materials

BIOLOGY FORM FOUR SUMMARIZED NOTES

BIOLOGY FORM TWO SUMMARIZED NOTES

Biology notes form 1-4; Free KCSE downloads

Free Biology notes, revision questions, KCSE past Papers, Exams, Marking Schemes, Topical revision materials, Syllabus and Many more

BIOLOGY FORM ONE NOTES FREE

BIOLOGY FORM THREE SUMMARIZED NOTES

Biology notes (Updated form one to four free notes)

Biology Simplified Notes Form 1 to 4 Free

Biology Free notes and Exams for Form one to Four

BIOLOGY KCSE MARKING AND SETTING TIPS IN LINE WITH EMERGING TRENDS

Biology topical questions and answers

Biology KCSE Past Papers and all marking schemes free downloads

Biology free lesson plans for all topics (Form one to four)

CBC Senior School Subjects (Grade 10, 11, 12)

CRE free lesson plans for all topics (Form one to four)

History and Government free lesson plans for all topics (Form one to four)

Free Secondary School Exams and Marking schemes (Form 1 to 4)

Syllabus For All Secondary Schools Per Subjects (Latest Syllabus)

English free lesson plans for all topics (Form one to four)

Free Physics notes, revision questions, KCSE past Papers, Exams, Marking Schemes, Topical revision materials, Syllabus and Many more

FORM 4 EXAMINATIONS AND MARKING SCHEMES: ALL SUBJECTS FOR KCSE CANDIDATES- OVER 1,000 PAPERS

Form 4 Term 1-3 Free Exams and marking schemes; All subjects downloads

FORM 3 ALL SUBJECTS EXAMS, ASSIGNMENTS: FREE TERM 1-3 EXAMS & ANSWERS

Updated Secondary School Notes form one to four Free Downloads (All subjects Comprehensive Notes)

Physics free lesson plans for all topics (Form one to four)

(KCSE 2002)

  • a). Describe the functions of the various parts of the human eye (20 marks; KCSE2003)

                                                                                                                                         1 | P a ge

(b). Describe how fruits and seeds are suited to their modes of dispersal (20 marks; KCSE 2003)

  • a). How is the mammalian skin adapted to its functions? (20 marks; KCSE2004)

b). Explain how a biotic factors affect plants (20 marks; KCSE 2004)

  • a). Describe how gaseous exchange takes place in terrestrial plants (20 marks; KCSE2005)

b). How is the human eye adapted to its function? (20 marks; KCSE 2005)

  • a). Describe how human kidney functions (20 marks; KCSE2006)

b).Describehowwatermovesfromthesoiltotheleavesinatree (20marks; KCSE2006)

  • a). Describe the structure and functions of the various parts of the human ear (20 marks; KCSE 2007 )

b). Describe causes and methods of controlling water pollution (20 marks; KCSE 2007 )

  • Describe the nitrogen cycle (20 marks; KCSE2008)
  • a). State four characteristics of gaseous exchangesurfaces

b). Describe the mechanism of gaseous exchange in a mammal (16 marks; KCSE 2008)

  • a). How are flowers adapted to wind and insect pollination? (20 marks; KCSE2009 )

b). Describe the role of the liver in homeostasis in the human body (20 marks; KCSE2009)

  • a). Describe the process of fertilization in flowering plants (20 marks; KCSE2010)

b). Describe how a finned fish such as tilapia moves in water (20 marks; KCSE 2010)

  • a). Describe the exoskeleton and its function in insects (13 marks; KCSE2011)

b). Describe how accommodation in the human eye is brought about when focusing on a near object (7 marks; KCSE 2011)

  • Using a relevant example in each case, describe simple and conditional reflex action (20 marks; KCSE2012)
  • a).Usingarelevantexample,describehowanallergicreactionoccursinahumanbeing

(10 marks; KCSE 2012)

b).Describehowenvironmentalfactorsincreasetherateoftranspirationinterrestrialplants

  • a). Describe the process of blood clotting in human beings (10 marks; KCSE2013 )

b).Howarerespiratorysurfacesinmammalsadaptedtotheirfunctions? (10marks; KCSE 2013)

  • Describe the role of the following organs in excretion andhomeostasis
  • The liver (10 marks; KCSE2013)
  • The skin during hot environmental conditions (10 marks; KCSE2013 )
  • a). Explain how each of the following factors affect the rate ofphotosynthesis:
  • Temperature (2 marks; KCSE2014)
  • Chlorophyll concentration (2 marks; KCSE2014 )

b). Describe the process of carbohydrate digestion in human beings (16 marks; KCSE2014)

  • a). How does excretion take place in plants (4 marks; KCSE2014)

b). Describe the role of the human skin in homeostasis (16 marks; KCSE 2014)

  • a).Explainthevariouswaysinwhichseedsandfruitsareadaptedtodispersal (20marks; KCSE 2015)

b). How is a mammalian heart structurally adapted to its function? (20 marks; KCSE2015)

                                                                                                                                         2 | P a ge

  • (a).Usingarelevantexampleineachcase,describesimpleandconditionalreflexaction

(20 marks; KCSE 2016)

b). Describe how the mammalian heart is structurally adapted to its function (20 marks; KCSE 2016)

  • a ) Explain the importance of protecting the forest ecosystem with reference to the following (20 marks; KCSE 2017 )
  • a) Climate change
  • b) Biodiversity

c)Biotechnology

d)Water conservation

  • e) Pollution
  • b) Describe how a mammalian eye is structurally adapted to its functions (20 marks)
  • a) Describe the mode reproduction in a named fungus (5 marks)
  • b) Describe the roles of hormones in the menstrual cycle (15 marks; KCSE 2018 )
  • a) Giving examples, describe the following among organisms (20marks; KCSE 2019 )
  • Predator-prey relationship
  • b) Explain the effect of increased physical activity on the following organ system (20 marks)
  • a)(i)Explain the role of the liver in blood regulation (3 marks ; KCSE 2020)
  • Describe how a mammalian heart is adapted to its functions (17 marks)
  • b) (i) Explain how the presence of chloroplast in guard cell affect the opening of the stomata (5marks)

(ii) Describe how various environmental factors affect the rate of photosynthesis (15marks)

  • a) (i) Explain the role of placenta during pregnancy(10 marks) ( KCSE 2021)
  • ii) Explain features and mechanisms that hinder self pollination and self fertilization(10 marks)

b)( i) Describe how xylem is structurally adapted to its functions  (5marks)

(ii) Describe the functions of mammalian blood in the human body(15 marks)

  • a i) Describe how plants eliminate waste products(8 marks; KCSE2022 )

ii)Describe the structure and function of mammalian nephron (12marks)

  • b) i. Describe five tropic responses and their survival values (15marks)

ii)Describe how mammalian heartbeat is controlled                (5mks)

Related Posts

Schemes of Work For Home Science Grade 7 Junior School CBC

Your email address will not be published. Required fields are marked *

Email Address: *

Save my name, email, and website in this browser for the next time I comment.

  • TSC Latest News
  • Teachers’ Resources
  • Knec Schools Portal
  • KUCCPS News Portal
  • Universities and Colleges
  • KMTC Portal
  • KSSSA School Games News
  • HELB News Portal
  • Teachers’ News
  • Advertise With Us

pep

Find what you need to study

Biology Short Essay Free Response Questions

10 min read • june 11, 2020

Jessica Nadzam

Jessica Nadzam

Attend a live cram event

Review all units live with expert teachers & students

Overview of the Short Essay (FRQ) Questions

The AP Biology exam used to consist of eight long-form free response questions, but in 2019 it was redesigned to consist of only six free response questions. Two of those questions are in the “short” form, and these consist of 50% of the free response score, or 25% of your total score. 

Long story short, those four questions may not be as long as the first two, but they still pack a big punch on your score.

The short essay questions will always be the last four on your exam. They’re very different than the first two in length and point-value, but you can use the same techniques as the others. You still need to know the content and utilize the “essential verbage” to write an appropriate response, but this response will be shorter and take less time to complete. 

You have to write in blue or black ink (for all things that are good, please do not use a pencil), so have a few of your favorite pens ready. You’ll have lots of pages to write on (if you use all of them, you may have written too much), and you’ll be given a copy of the Formulas and Equations Sheet for any calculations you need to do. 

Questions are 4 points apiece, and they typically contain content and problems concerning multiple content areas. Since they are shorter they are less likely to cover multiple topics, but it does still happen. You could initially respond about protein structure in the first part and then end up describing the evidence of evolution by the end of the question. There’s a lot of points on the table, and CollegeBoard will make you work for them. 

Response Grading (from an AP Reader)

There are 16 points for the last four questions of the FRQ test, and they make up 25% of your AP Biology score. So, how are those points calculated and scored? CollegeBoard is fantastic at writing rigorous questions, but they’re also very good at training their graders (called AP Readers) how to objectively score the questions. Readers spend a whole day training to grade just two of the questions (they don’t get to look at the other four at all), and then spend a whole week grading only those questions, eight hours a day . 

A well-oiled machine may be an understatement. The bottom of the line is, AP Readers know exactly what they’re looking for when they’re grading. They read a hundred tests a day (minimum), and if you don’t have the answer they’ve been told is the right answer, they tend to move on pretty quickly. 

While this seems intimidating, it’s actually good news for you - but only if you pay very close attention to the next section on verbiage. You see, the bolded verbs in AP free response questions are just hints as to what AP Readers are looking for in an answer. If you know those verbs backwards and forwards, you will know what type of response AP Readers want. And if you know that, you don’t have to worry about writing something that may be right content-wise, but still missing a point on a technicality. 

Essential Verbiage

There are a lot of verbs used in AP exams to pose questions for students to ponder and rip their hair out over. 

To keep all that hair on your head, we’ve made you a table. It’s pretty easy - if you know exactly what to do when you encounter each bolded verb , you know exactly what to do to answer the question the way the Reader wants to see it. You know how many points it’s worth, and therefore can figure out how much time to spend on it. 

Side note - keep in mind that if a question asks you do something more than once - maybe to describe two factors or explain three phenomenon, you should multiple that # Points by however many things it expects you to do. 

https://firebasestorage.googleapis.com/v0/b/fiveable-92889.appspot.com/o/images%2F-nnXhfta0BGOb.png?alt=media&token=42104a92-a2e6-4cf0-9f13-c25fb4d0a231

Strategies for 5able Responses

Do you want a 5 on this test? Then here’s what you need to know and be able to do:

Read. The. Question. Carefully. Seriously. You don’t know how many students miss points because they were skimming and missed a critical word. It’s a heartbreaker for Readers, who want to give you that point, but can’t. Also, read the directions carefully. Your papers always say something about how responses on the actual question page will not be scored. Do not write on the question page . You can use it for brainstorming or outlining as scratch paper, but if your words aren’t written on lined paper or a graph or table, they won’t be scored. So make sure you put your answers in the right place! 

Complete. Sentences. Unless it’s an identify or construct or calculate question, you need a capital letter and a period, or the Reader will not grade that response. Period. Outlines, bulleted lists, and drawings won’t be graded unless specified in the question’s directions. 

Circle your bolded verbs . Find them. Figure out which are worth the most points, the least points, will take the longest, will be the quickest, etc. Then, figure out which ones to answer first. 

When you start the test, you don’t have to go in order. If your mind goes blank on question one, skip it . It’s not going anywhere, you can come back to it. But don’t waste time on it when there are others you can definitely answer later in the booklet. 

The identify and graph/draw questions are some of the quickest to respond to because they don’t require complete sentences, and the answers are usually very quick to come up with. Answer these questions first to get them out of the way. 

Time yourself. You get 90 minutes, but it goes by fast. Keep a digital watch that does not beep , and refer to it regularly. Plot how long you want to spend on each question so you don’t waste time in one area and lose points on another question you could have easily answered. Since the short questions are worth half your free-response score, you shouldn’t spend any more than 45 minutes on these four questions, or about 10-11 minutes apiece. 

This is not AP English. The graders are not English teachers. They don’t care how pretty your introduction is or how ~thought-provoking~ your thesis statement is. They just want to finish your paper and move on, so don’t ramble on and on. Skip the fluff and go straight to the point. Don’t restate the question or introduce the topic or regurgitate random knowledge - it won’t get you extra points, it’s a waste of time for you, and the Readers get bored sorting through all your thoughts and writing. 

Label your responses . While you still have to write in complete sentences, please label each response with a, b, c, etc. if the essay question has multiple parts. You don’t even have to go in order, but this helps Readers find your answer so they don’t have to sift and guess what you meant.

COMMIT TO YOUR ANSWER . Readers don’t like wishy-washy papers. Erase the words “might” and “possible” and “I think” from your vocabulary. You could be completely wrong, but if you’re writing about a purple hippopotamus, COMMIT to that purple hippopotamus. Don’t say “I think the purple hippopotamus might possibly maybe do photosynthesis if it feels like it.” That’s wishy washy, and Readers don’t accept that as an actual answer, so they won’t give points for it. Be committed , and loudly proclaim “This purple hippopotamus does photosynthesis!” Ta-da! 

If possible, always give an example. We said earlier not to mind dump and regurgitate, but examples are usually a good way to sweep up an extra point or two if you have budgeted your time wisely and can accurately apply it to the scenario. 

So, now that you know what the Readers are looking for in your responses, let’s talk about the types of questions they will ask you. 

Question Types

The short version of the free-response test has four questions, and each of those four questions has a distinct question type. These are based on “science practices” that CollegeBoard expects students to develop to be more ~well-rounded~ and ~critical thinkers~. Luckily, these questions go in a distinct order, too. Those types and order is as follows:

Scientific Investigation 

Conceptual Analysis

Analysis of Model or Visual Representation 

Analysis of Data 

With this, you can get a good idea of what to expect on the 3rd, 4th, 5th, and 6th question. And if you’re used to studying for the science section of the ACT, you probably recognize these question types. It’s all about analyzing graphs and tables, pulling information from passages, and performing analysis on data. 

Oh, while remembering all that AP Biology knowledge your teacher tried to get into your brain for the past 6-9 months. No pressure. 

Let’s break down what each of those question types is asking for, and the best ways to respond to maximize your time and your points. 

Scientific Analysis - this question will describe a lab experiment scenario, and expect you to be able to describe the biological concept or process involved, identify an experimental procedure, predict results, and justify your prediction.

Conceptual Analysis - in this question, CollegeBoard will present a question that describes a real-life scenario. It will relate that scenario to a biological phenomenon and tell you something has disrupted the process. To get all four points, you will have to describe the biological process that is happening, explain that biological concept, predict the causes and effects of the disruption, and justify your prediction. 

Analysis of Model or Visual Representation may seem like two separate question types, but they will ask you to do the same thing. Both will give you a description of some biological scenario with a visual model. You will be asked to analyze that model, and then describe different biological characteristics of the model, explain the relationships between the different characteristics, identify or draw the relationships within that model, and then explain how the scenario relates to a larger idea or concept. 

Analysis of Data - for the final question of the exam (and then you’re freeeeee !), you will receive data on a graph or table. You will then have to describe the data (twice), evaluate a hypothesis or prediction using the data, and then explain how the experimental results relate to some biological concept. 

These descriptions can seem a bit confusing on their own. Therefore, some examples are provided below. 

Sample Question 1 - Scientific Investigation  

A forest ecosystem has a delicate balance of life. Autotrophs, heterotrophs, and decomposers live together and exchange resources to survive. Sometimes their relationships are mutually beneficial, but sometimes they can be harmful for one species and beneficial for another. Buteo jumaicensis , or the red-tailed hawk, consumes smaller organisms such as small reptiles and small mammals. Recently, the ecosystem has been disturbed by deforestation of pine trees. Hawks typically roost on the crowns of tall trees, and as a result their habitat begins to diminish. 

a) Describe the niche of the red-tailed hawk. 

b) Identify the type of relationship shared between the red-tailed hawk and a common forest mouse. 

c) Predict the short-term impact of the decline of the red-tailed hawk due to deforestation, and how it will impact the rest of the ecosystem. 

d) Provide reasoning to justify your prediction. 

Sample Question 2 - Conceptual Analysis  

The lac operon model is a common phenomenon used to demonstrate the regulation of gene expression. In this model, a gene is essentially turned “on” and “off” to produce proteins only when they are necessary. This allows a bacteria called Escherichia coli ( E. coli) to metabolize lactose. 

https://firebasestorage.googleapis.com/v0/b/fiveable-92889.appspot.com/o/images%2F-rS3kbRK6a5fR.png?alt=media&token=a3fb37d9-bde3-4909-902d-f0666d00a78b

a) Describe the gene expression process from DNA to protein. 

b) Explain why operons are an evolutionary advantage over allowing genes to undergo transcription and translation constantly. 

c) Although less commonly used for demonstration, the trp operon is another bacterial operon found in E. coli. Predict what would occur if a lac repressor was used on the trp operon. 

d) Provide evidence to support your prediction. 

Sample Question 3 - Analyze Model or Visual Representation  

Signal transduction pathways occur when cells receive signals from outside their membranes, and transduct those messages through the cell to a predetermined location. These signals are used for a variety of reasons including growth, division, mating, protein production, and more. In order for a cell to experience signal transduction, it must receive some signal that binds to a receptor and triggers the pathway. 

https://firebasestorage.googleapis.com/v0/b/fiveable-92889.appspot.com/o/images%2F-1vIz8wwFuab0.png?alt=media&token=9a27a73a-a18c-4ed4-b4b5-398b1ed7245f

a) Referring to the figure, predict a possible response that may occur as a result of this signal. b) Describe the steps of the signal transduction pathway that must occur to elicit a response. 

c) Sometimes secondary messengers may enter the cell only after a ligand has bound to a receptor. Explain why secondary messengers may be necessary for cell signalling. 

d) Identify the ligand in the figure. 

Fiveable

Stay Connected

© 2024 Fiveable Inc. All rights reserved.

AP® and SAT® are trademarks registered by the College Board, which is not affiliated with, and does not endorse this website.

in the light of the science!

  • Planet Earth
  • Strange News

How To Answer Biology Essay Questions

Table of Contents:

What Exam Skills are Required to Pass AQA Biology? . We all know that a well designed and rigorous revision schedule is necessary to pass any exam. Here are some tips to revise for Biology.

Before the exam: Try and relax in the last half-an-hour or so before the exam. Read a book, sit quietly somewhere, listen to music – whatever works for you. Just don’t do any last-minute revision. It almost certainly won’t stay in your head anyway. So long as you’ve revised well in the weeks leading up to the exam you should be OK.

Video advice: How to write a good biology essay by Mr. Wanyama

Subscribe to our channel

How To Answer Biology Essay Questions

Tips on Answering AQA Biology Exam Questions – Be clear and concise in your answers. For diagrams, don’t fuss too much over perfect accuracy but make sure that all the key points are there, in the right place, and clearly labelled. For “Brief notes” questions, bullet-point sentences containing relevant information will do as well as, if not better than, a brief essay where the information required is buried under too many words. If you are asked to show your working, do so clearly, making use of the available space to answer the question. Don’t skip too many lines of working at once.

KCSE 2022 Biology Essay Questions and Answers (KCSE 2022 Prediction Questions)

Here are KCSES 2022/2022 Biology Essay Questions and Answers (KCSE 2022 Prediction Questions). Content: 31 pages with 60 questions and answers.

Needed at nighttime stage of photosynthesis it combines using the hydrogen ion in the light stage to create glucose, proteins and lipids low concentrations cuts down on the rate of manufacture of energy and food while high concentrations results in a rise in the quantity of energy and food formed

BIOLOGY (231/2) Revision Questions (Essays): Expected Responses

Has a cell membrane; with pores; that regulates substances entering and leaving the cell; cytoplasm; contain sugars and salts; for maintaining its osmotic pressure; also has a liquid medium; for all biochemical reactions; nucleus; contain chromosomes having hereditary material; and controls all the activities of the cell; ribosomes; are sites for protein synthesis; golgi bodies/apparatus; for secretion of hormones and enzymes; formation of lysosomes; lysosomes; contain lytic enzymes for breaking down worn-out organelles; secretory vesicles; formed from golgi apparatus for secreting substances; smooth endoplasmic reticulum; synthesizes and transports lipids; rough endoplasmic reticulum; transport proteins; nucleolus; controls the activities of the nucleus; produces ribosomes; mitochondria; form sites for energy production; centrioles; formation of cilia and flagella; forms spindle fibres used in cell division; plant sap vacuoles; store salts and other dissolved substances; controls osmotic pressure and turgidity of cells; food vacuoles involved in digestion of engulfed food; chloroplasts; form sites for photosynthesis in plant cells; Max.

Video advice: How to get TOP MARKS in a biology essay: AQA A-level 25 mark essay on paper 3

Learn how to write the 25 mark essay on the AQA A-level paper 3. I fully explain the mark scheme, how to analyse the titles, how to structure your paragraphs and how to write a top plan for success. I model the plan for ‘The Importance of Diffusion’ 2017 title.

How To Answer Biology Essay Questions

AnsweringEssayQuestions

COMMUNICATION IN THE BIOLOGICAL SCIENCES Department of Biology.

Don’t simply create a drawing and expect the teacher to determine that which you were thinking out of this. (Unless of course the issue only insists upon create a drawing. ) Don’t expect the teacher to obtain the relevant information inside a ocean of irrelevant information. Don’t expect the teacher to see between your lines making connections that you ought to make.

The question will always involve two or more related items. “Compare” means that you should explain the similarities between the two items. Ordinarily, instructors do not want you to simply list the similar characteristics, but explain the characteristics and/or how they are similar. “Contrast” means that you should explain the differences between the two items. Typically, a comparison of the similarities and differences between the two items highlights some major concepts in the topic at hand. Be sure to try to address these in your answer. This type of question usually involves the use of specific examples from class.

Biology Form 4 Kssm Questions And Answers – Fill Biology Form 4 Kssm Questions And Answers, Edit online. Sign, fax and printable from PC, iPad, tablet or mobile with pdfFiller ✔ Instantly. Try Now!

Video advice: HOW TO WRITE EXAM ESSAYS! UNIVERSITY BIOLOGY STUDENT TIPS + ADVICE

HOW DO I WRITE MY EXAM ESSAYS? ��⏰

How To Answer Biology Essay Questions

How do you answer an essay question?

6:509:355 Rules for Answering ESSAY Questions on Exams - YouTubeYouTubeStart of suggested clipEnd of suggested clipDon't restate the prompt in your introduction. Instead write an interesting thesis statement thatMoreDon't restate the prompt in your introduction. Instead write an interesting thesis statement that covers the prompt. But in your own words. And finally ensure your conclusion. Synthesizes.

How do you write a short answer question in biology?

2:018:53How to answer short response science questions - YouTubeYouTubeStart of suggested clipEnd of suggested clipThe verb in the first question is described in the verb in the second sentence is explained. TheseMoreThe verb in the first question is described in the verb in the second sentence is explained. These are two different words that require you to do two different things in your answer.

How do you write a biology essay?

You should create a good first impression regarding your comprehension about the topic of your essay in the introduction section.

  • State and elaborate on the topic of your paper.
  • Outline a brief literature review of the topic.
  • Outline the parameters you've used in your paper.
  • State your main points and arguments.

How do you write a long answer in biology?

5 Exam tips for Biology

  • Structure your answers efficiently. There are some easy traps to fall into when it comes to crafting an answer in your Biology exam. ...
  • Draw if it's easier. ...
  • Do the multiple-choice questions first. ...
  • Figure out the key terms in the question. ...
  • Highlight the key terms in your answer.

What is the proper way to answer exam questions?

Strategies for answering exam and test questions

  • Read through the options and try to eliminate the ones that aren't right. ...
  • Don't struggle over a question. ...
  • Answer all the questions. ...
  • When you check back through your paper and think an answer is wrong - change the answer.

Related Articles:

  • How To Answer Gcse Science Questions
  • How To Answer Physics Practical Questions
  • How To Answer Science Exam Questions
  • Science Using the Webb Space Telescope – What Questions Does It Answer?
  • Want the very best Look at Comet NEOWISE? NASA Experts Discuss and Answer Public Questions
  • Physicists Uncover Strategies of World’s Thinnest Superconductor – Answer 30-Year-Old Questions

biology essay questions and answers 2022

Science Journalist

Science atlas, our goal is to spark the curiosity that exists in all of us. We invite readers to visit us daily, explore topics of interest, and gain new perspectives along the way.

You may also like

Do We Know More About Space Than The Ocean

Do We Know More About Space Than The Ocean

What Does Volatile Mean Chemistry

What Does Volatile Mean Chemistry

What Innovation Was Brought By The Ag Revolution

What Innovation Was Brought By The Ag Revolution

Add comment, cancel reply.

Your email address will not be published. Required fields are marked *

Save my name, email, and website in this browser for the next time I comment.

Recent discoveries

How Much Water Covers The Earth In Fractions

How Much Water Covers The Earth In Fractions

How To Free Up Space On Nintendo Switch

How To Free Up Space On Nintendo Switch

What Is The Oldest Living Thing On Earth

What Is The Oldest Living Thing On Earth

Where Is Science Taking Us In Punjabi

Where Is Science Taking Us In Punjabi

  • Animals 3041
  • Astronomy 8
  • Biology 2281
  • Chemistry 482
  • Culture 1333
  • Health 8466
  • History 2152
  • Physics 913
  • Planet Earth 3239
  • Science 2158
  • Strange News 1230
  • Technology 3625

Random fact

How To Write Science In Bubble Letters

How To Write Science In Bubble Letters

AP Biology Practice Exams

We have links to all of the best online AP Biology practice exams. These resources will provide thousands of challenging practice questions to work through. Choose from the listing below to get started with your test prep right now!

AP Biology Practice Test

Varsity tutors, biology (mader) 8th edition, ap biology math review, official practice test, college board 1999 exam, albert ap biology practice, biology (mader) 10th edition.

AP Biology | Practice Exams | Free Response | Notes | Videos |  Study Guides

Woolton Tutors

Woolton Tutors

One-to-one tutoring in science, maths and languages

Woolton Tutors

How to answer A-level Biology essay questions

How to answer A-level biology essay questions

The Synoptic essay questions in paper 3 of the AQA A-level biology course carries 25 marks, so can make a big difference to your overall grade. The essay encourages you to think across different topics (some call this “synoptic” or “holistic” thinking). This is also valuable for other areas of the exams, particularly the application questions .

Here are a few tips for tackling your A-level biology essay: (As an example, Let’s use the title “The importance of movement in cells and tissues”)

Think broadly…

Essay titles are deliberately vague to give you the chance to show your knowledge in a variety of topics. To help you to choose what’s most relevant, look out for subjective words like “movement” and “cells”. Think of the possible alternatives. “Cells”, for example, hints that you could discuss plants, animals, single-celled organisms etc. in your essay. “Movement” could mean short distances (across a membrane) or much further (circulation or mass flow) or even the whole tissue moving (phototropism).

Visualize and catastrophize!

If you can’t think of enough topics for an “importance” essay, flip the question around – what would happen if movement in cells suddenly ground to a halt? Or if ATP suddenly vanished, or our cells were robbed of their ions? Sometimes catastrophizing in this way frees up thoughts, and helps you see what is really important. Another way is to picture every diagram in your notes or textbook that features the thing-that’s-important-in-the-question. If it’s in a diagram, it could go in your essay.

Pick your strongest topics

It’s possible to drop marks on the essay for scientific errors. With that in mind, even if you have lots of ideas for things to include, always pick the topics you can write confidently about. For bonus marks, try to order them in a way that “flows” like a story. Take care to use A-level (and not GCSE) language too.

Keep an eye on the question

While you’re writing, keep asking yourself “Where am I placing the emphasis?”. If the essay is about movement, perhaps you don’t need a whole page about nervous responses, only the bits where movement is involved – sodium ions flowing through channels, waves of depolarisation etc. Movement is the focus of the essay, after all, not neurons. Always ask yourself “Does this sentence fit with the title of the essay?”

Use outside knowledge

The top marks for the essay are reserved for displaying “evidence of reading beyond specification requirements”. Often you can drop recent scientific research or news into your writing – how vaccines tackle COVID might work in an essay about viruses, for example. I often encourage students to have a quick look at the news pages of New Scientist pages (eg https://www.newscientist.com/subject/health/ ) before paper 3 for inspiration – you don’t need to spend ages doing this, just long enough to have one or two ideas up your sleeve…

Good luck with your essay!

If you’d like to work through some example essay questions, please get in touch with me at Woolton Tutors (or Woolton Biology) and we can set up some online biology tutoring sessions.

Best wishes,

Dr John Ankers

Specialist online A-level biology tutor

www.wooltontutors.co.uk

Share this:

' src=

Author: Dr John Ankers

Dr John Ankers is a tutor, coach and writer. For writing and consultancy work, please contact me at [email protected] View all posts by Dr John Ankers

Leave a Reply Cancel reply

Your email address will not be published. Required fields are marked *

Notify me of follow-up comments by email.

Notify me of new posts by email.

SCIENCE ORBIT

Search this blog.

biology essay questions and answers 2022

BIOLOGY ESSAY COLLECTION Compiled By: A.L.Miskeen (MSc PGDE SLPS)

biology essay questions and answers 2022

You might like

Post a comment, contact form.

Easyelimu Logo

  • Form 1 Mathematics Notes
  • Form 2 Mathematics Notes
  • Form 3 Mathematics Notes
  • Form 4 Mathematics Notes
  • Form 1 Mathematics Topical Questions and Answers
  • Form 2 Mathematics Topical Questions and Answers
  • Form 3 Mathematics Topical Questions and Answers
  • Form 4 Mathematics Topical Questions and Answers
  • Form 1 Functional Writing Notes
  • Form 2 Functional Writing Notes
  • Form 3 Functional Writing Notes
  • Form 4 Functional Writing Notes
  • Poetry Notes
  • Grammar Notes
  • Oral Literature Notes
  • Oral Skills Notes
  • Guide to Blossoms of the Savannah Summarized Notes - Easy Elimu
  • A Doll's House
  • The Pearl Study Guide
  • Memories We Lost and Other Stories Study Guide
  • Inheritance Study Guide
  • A Silent song and Other Stories Guide
  • Fathers of Nations Guide
  • An Artist of the Floating World Guide
  • The Samaritan Guide
  • Sarufi na Matumizi ya Lugha
  • Isimu Jamii Notes
  • Fasihi Notes
  • Ushairi Notes
  • Mwongozo wa Kuandika Insha
  • Tumbo Lililoshiba na Hadithi Nyingine
  • Mwongozo wa Kigogo
  • Mwongozo wa Chozi La Heri - Chozi la Heri Notes PDF
  • Mwongozo wa Bembea ya Maisha - Bembea ya Maisha Notes PDF
  • Mwongozo wa Nguu za Jadi
  • Mwongozo wa Mapambazuko ya Machweo na Hadithi Nyingine
  • Biology Form 1 Notes
  • Biology Form 2 Notes
  • Biology Form 3 Notes
  • Biology Form 4 Notes
  • Biology Essays
  • Form 1 Biology Topical Revision Questions and Answers
  • Form 2 Biology Topical Revision Questions and Answers
  • Form 3 Biology Topical Revision Questions and Answers
  • Form 4 Biology Topical Revision Questions and Answers
  • Form 1 Chemistry Notes
  • Form 2 Chemistry Notes
  • Form 3 Chemistry Notes
  • Form 4 Chemistry Notes
  • All Chemistry Practicals Notes for KCSE and MOCKS
  • Form 1 Chemistry Topical Revision Questions and Answers
  • Form 2 Chemistry Topical Revision Questions and Answers
  • Form 3 Chemistry Topical Revision Questions and Answers
  • Form 4 Chemistry Topical Revision Questions and Answers
  • IRE Form 1 Notes
  • IRE Form 2 Notes
  • IRE Form 3 Notes
  • IRE Form 4 Notes
  • Physics Form 1 Notes
  • Physics Form 2 Notes
  • Physics Form 3 Notes
  • Physics Form 4 Notes
  • CRE Form 1 Notes
  • CRE Form 2 Notes
  • CRE Form 3 Notes
  • CRE Form 4 Notes
  • Geography Form 1 Notes
  • Geography Form 2 Notes
  • Geography Form 3 Notes
  • Geography Form 4 Notes
  • History Form 1 Notes
  • History Form 2 Notes
  • History Form 3 Notes
  • History Form 4 Notes
  • Business Studies Form 1 Notes
  • Business Studies Form 2 Notes
  • Business Studies Form 3 Notes
  • Business Studies Form 4 Notes
  • Home Science Form 2 Notes
  • Home Science Form 3 Notes
  • Home Science Form 4 Notes
  • Home Science Form 1 Notes
  • Agriculture Form 1 Notes
  • Agriculture Form 2 Notes
  • Agriculture Form 3 Notes
  • Agriculture Form 4 Notes
  • Agriculture KCSE 2019 Project
  • Computer Studies Form 1 Notes
  • Computer Studies Form 2 Notes
  • Computer Studies Form 3 Notes
  • Computer Studies Form 4 Notes
  • KCSE 2017 Reports
  • 2018 Pre-Mocks
  • 2019 Pre-Mocks
  • 2022 Pre Mocks
  • 2021/2022 Pre-Mock Past Papers
  • 2023 Pre Mocks
  • 2017 Mock Past Papers
  • 2019 Mock Past Papers
  • 2020 Mock Past Papers
  • Mock Exam Papers 2021/2022 - Easy Elimu
  • Mock Exam 2022 Questions and Answers
  • Alliance Boys High School
  • Maranda High School
  • Form 1 Past Papers
  • Form 2 Past Papers
  • Form 3 Past Papers
  • Form 4 Past Papers
  • 2019 KCSE Prediction Papers
  • 2020 KCSE Prediction Papers
  • 2021 KCSE Prediction Papers
  • 2022 KCSE Prediction Questions and Answers - EasyElimu
  • KCSE Prediction 2023
  • 2020 Post Mock Past Papers
  • 2021/2022 Post Mocks
  • 2023 Post Mocks
  • Play Group: Activities, Homework and Syllabus
  • 2023 PP1 Exams
  • 2023 PP2 Exams
  • Grade 1 Notes
  • 2023 Grade 1 Exams
  • Grade 2 Notes
  • 2023 Grade 2 Exams
  • Grade 3 Notes
  • 2023 Grade 3 Exams
  • Grade 4 Notes
  • 2023 Grade 4 Exams
  • Grade 5 Notes
  • 2023 Grade 5 Exams
  • Grade 6 Notes
  • KPSEA Exams
  • 2023 Grade 6 Exams
  • Class 6 : Notes, Revision Papers and Syllabus
  • Class 7 : Notes, Revision Papers and Syllabus
  • Class 8 Notes
  • 2023 Class 8 Exams
  • 2023 Kcpe Prediction
  • Grade 7 Notes
  • 2023 Grade 7 Exams
  • Pre Mock Exams 2024
  • Form 4 Term 2 Opener Exams 2024  
  • Form 3 Exams 2024
  • Form 2 Term 2 Opener Exams 2024
  • Form 1 Term 2 Opener Exams 2024
  • All Kiswahili setbook guides
  • All English setbook guides
  • Form 1 - 4 High School Notes

Biology Paper 1 Questions and Answers - KCSE 2022 Mock Exams Set 2

« Previous Topic Physics Paper 3 Questions and Answers with Confidentials - KCSE 2022 Mock Exams Set 2

Next Topic » Biology Paper 2 Questions and Answers - KCSE 2022 Mock Exams Set 2

INSTRUCTIONS

  • All Questions are Compulsory
  • Wrong Spelling of Technical Terms shall be Penalized
  • State TWO ways in which the study of Biology has helped the world in the accelerated fight against the recent Covid-19 pandemic. (2mks
  • Give ONE function of centrioles (1mk
  • Name a Kingdom in which all members lack centrioles in their cells (1mk)
  • Name the skin pigment formed by cells in the mammalian skin
  • Which genetic disorder is associated with the absence of the pigment named in 2a)?
  • How does the skin pigment help protect human beings against skin cancer?(1mk)
  • More water hyacinth plants are found growing along the shore of Lake Victoria than in the deep waters (1mk)
  • Green plants grow faster in lower altitudes areas than in higher altitude areas (2mks)
  • State TWO functions of a cover slip in light microscope work (2mks)

Biocsp1q6

  • From the diagram, give a reason to show that X is the left ventricle (1mk)
  • Name a class of organisms where all members have the heart structure above(1mk)
  • Why are the muscles found in the heart above said to be myogenic? (1mk)
  • Red blood cells lack mitochondria (1mk)
  • The testes are found hanging outside the body in male human beings (1mk)
  • Why is pancreas said to be a dual gland? (1mk)
  • Name the endocrine tissue in the pancreas (1mk)

Biocsp1q9

  • In which beaker did the dialysis tubing reduce in size? (1mk)
  • Account for your answer in a) above (2mks)

Biocsp1q10

  • State the name used to refer to animal A in terms of thermoregulation (1mk)
  • State TWO advantages animal B has over animal A (2mks)

Biocsp1q11

  • Name the agent of pollination for this flower (1mk)
  • Give TWO adaptive features from the diagram to support your answer in a) above (2mks)

Biocsp1q12

  • State a reason why the process above represents gamete formation in female mammals (1mk)
  • Name the chromosomal mutation represented above (1mk)
  • What was the role of pyrogallic acid in this experiment? (1mk
  • Determine the percentage of Carbon (IV) oxide in the sample of air (2mks
  • Is this sample of air exhaled air or inhaled air? (1mk

Biocsp1q14

  • Give a reason why the diagram above represents hypogeal germination(1mk
  • Account for the change in dry weight of the cotyledon at stage X and Y(2mks

Biocsp1q15

  • Acrosome in the spermatozoa(2mks)
  • Hair-like structures in the fallopian tube(1mk)
  • What are vestigial structures?(1mk)
  • Explain why divergent evolution is advantageous to living organisms?(2mks)

Biocsp1q18

  • What shows that the process represented above is taking place in a plant cell?(1mk

Biocsp1q19

  • Use arrows to show direction of impulse in a reflex arc in the diagram above (1mk
  • Outline expected TWO structural differences between nerve cell R and X(2mks
  • Cardiac sphincter muscles(1mk
  • Erector pili muscles(1mk

Biocsp1q21

  • Under which light condition was the experiment carried out.(1mk
  • Which term describes the appearance of the seedlings at the end of the experiment(1mk
  • What is the importance of the above experiment in crop production?(1mk
  • Palisade.(1mk
  • Schwann(1mk
  • Sertoli(1mk
  • Mary suspects that she has diabetes mellitus. Using a sample of her urine describe a school laboratory procedure she can follow to confirm it is true(3mks
  • Why is insulin not administered orally?(1mk

Biocsp1q24

  • Give the name of the bone.(1mk
  • State an advantage of the joint formed at the part labelled M(1mk
  • State the function of the part labelled N.(1mk
  • Distinguish between population and community as used in ecological studies(2mks
  • State the formula for population estimation using capture recapture method(1mk

Biocsp1q26

  • Name the type of cell division that formed the parts labelled A(1mk)
  • State the role of part labelled B(1mk)

Biocsp1q27

  • Name the class to which the organism shown above belongs.(1mk)
  • Give TWO reasons for your answer in a) above(2mks)

Biocsp1q28

  • Account for the expected result in the level of water in the beaker labelled B.(2mks)
  • Why were the set up C and D included in the experiment?.(1mk)

Marking Scheme

  • Knowledge on its cause; Knowledge on how the virus spreads; Increased knowledge on how to control the spread/containment measures; development of vaccines; global cooperation in research/fight against the virus; Mark 1st 2
  • Formation of Spindle fibers; Formation of flagella/cilia;
  • Plantae; Monera;
  • Absorbs UV/Ultraviolet rays;
  • Eutrophication/Enrichment of water by pollutants;
  • Higher concentration of Carbon (IV) Oxide; thus higher rate of photosynthesis; Conducive/Optimum temperatures; thus faster rate of metabolism\Activated enzymes;
  • Exclude dust/contaminants; Prevent dehydration; Hold specimen in position; OWTTE
  • Thicker muscular wall;
  • Mammalia; Aves;
  • Contract without nervous stimulation/Initiate contraction on their own;
  • To prevent oxygen utillisation for efficient transport;
  • To Create conducive/optimum temperature for sperm formation/ spermatogenesis;
  • Has endocrine and exocrine function/Forms both/ hormones and enzymes;
  •  Islet of Langerhans;
  • Sucrose solution is hypertonic; thus it gains water molecules by osmosis; Accept: Water is hypotonic; thus water molecules move into the dialysis tubing by osmosis;
  • Poikilotherm; Accept Ectotherm/Ectothermic/Poikilothermic Rej cold blooded
  • Can live in any habitat; Is More active both day and night;
  • Long filament; Anthers hanging outside; Feather/Net-like stigma; Smaller bracts to allow the stigma/anthers to be exposed to wind;
  • Only X chromosomes are involved;
  • Non-disjunction;
  • Turner’s Syndrome;
  • Klinefelter’s Syndrome;
  • Remove Oxygen;
  • % of CO 2 =[(7cm 3 – 6.8cm 3 ) X 100] ÷ 8cm 3 ; Deny if Units Omitted = [0.2 cm 3 X 100] ÷ 8cm 3 = 2.5%; Deny if % symbol missing
  • Exhaled air;
  • Cotyledon is found below the ground surface; elongated epicotyl;
  • Decreases; since stored nutrients are hydrolyzed to facilitate growth/germination/cell division;
  • 3cells measure 1.5mm Therefore 1 cell will measure? = (1cell X 1.5mm)/3cells; =0.5mm Deny if Units Omitted Magnification = image length/actual length Thus Actual length =image length/magnification =05mm/1000; Deny if Units Omitted =0.0005mm; Deny if Units Omitted
  • Release lytic enzymes; that Neutralize secretions/destroy/digest vitelline and cell membrane for entry of sperm head into the ovum;
  • Waft/push the ovum/zygote/blastula towards the uterus;
  • Structures that have lost function thus become smaller;
  • Development of necessary adaptations; to survive a give habitat/conquer different habitats;
  • D-Telophase;
  • Laying of a Cell plate/Cellulose deposits to separate the daughter cells; Centrioles/Spindle fibers are not involved/seen;

Biocsp1qa19

  • Cardiac sphincter muscles: Relaxes to allow entry of food/food bolus into the stomach/Prevent exit of food from stomach to oesophagus/gullet;
  • Erector Pili Muscles: Contracts and relaxes to make the body hairs straighten or lie flat on the body;
  • Low/dim; Accept darkness;
  • Etiolation;
  • Increase productivity of Dwarf plants;
  • Photosynthesis;
  • Secretion of myelin sheath;
  • Nourish young sperm cells; point of attachment of sperm cells as they develop;
  • Place 2ml of her urine in a boiling tube Add Equal amount/2ml of Benedict’ solution; Boil; Green/Yellow/Orange/Brown colour change would show a positive test/result;
  • It may be digested (since it is protein in nature);
  • Innominate bone/Half pelvic girdle; REJ Pelvic girdle alone
  • Achieves movement in all planes; Achieves a 3600 rotation thus greater flexibility;
  • Passage of blood vessels/nerve fibers to lower regions of the hind limbs; Reduces weight/density at the hip region;
  • Population is a group of organisms of same species found in the same place at the same time; while Community is different populations/group of animals and plants living in the same habitat; Award 0, 1 or 2 mrks
  • Population = [First Marked X Second Capture] ÷ Marked Recaptured REJ Abbreviations
  • Regulate Growth of pollen tube;
  • Mammalia; REJ Mammals/mammmalia/Mammmalian
  • Body fur/hair; Heterodont dentition/Different types of teeth; Presence of Ear pinna; 1st 2
  • Remained the same; all the stomata blocked by jelly thus no water uptake/loss/transpiration;
  • Act as a Control experiment;

Download Biology Paper 1 Questions and Answers - KCSE 2022 Mock Exams Set 2 .

Why download.

  • ✔ To read offline at any time.
  • ✔ To Print at your convenience
  • ✔ Share Easily with Friends / Students

Related items

  • Electricity Paper 2 Questions and Answers - KCSE 2021 Past Papers
  • Biology Paper 3 Questions - Kapsabet Boys Post Mock 2023 Exams
  • Biology Paper 2 Questions - Kapsabet Boys Post Mock 2023 Exams
  • Biology Paper 1 Questions - Kapsabet Boys Post Mock 2023 Exams
  • Biology Paper 3 Questions and Answers With Confidential- Kenya High Post Mock 2023 Exams

biology essay questions and answers 2022

access all the content at an affordable rate or Buy any individual paper or notes as a pdf via MPESA and get it sent to you via WhatsApp

What does our community say about us?

  • KCSE Revision Questions
  • Privacy Policy
  • Mobile App Privacy Policy
  • High Schools in Kenya
  • Teacher Resources
  • Questions and Answers
  • Online Tuition and Classes in Kenya

Copyright © 2022 EasyElimu

JiLearners.com

#1 Student Resource Platform

WAEC Biology Questions and Answers 2022/2023 Expo, Theory & OBJ

Free WAEC Biology questions and answers 2022/2023, WAEC biology expo 2022, WAEC biology runs for biology paper 1 and paper 2, biology theory, and OBJ papers 2022.

Do you need free WAEC Biology questions and answers 2022/2023 expo for Biology theory and OBJ papers? If yes, then here is something that will help you.

For you to make ‘A’, ‘B’, or at least ‘C’ in biology in WAEC 2022, you must work your way out to get the necessary expos that will help you pass the WAEC biology exam 2022.

This WAEC biology expo 2022 is presented in different formats for WAEC biology theory questions and answers 2022 and WAEC biology objective (OBJ) questions and answers 2022.

Table of Contents

WAEC Biology Examination Date and Schedule for 2022

The 2022 WAEC biology exam is scheduled to hold as follows:

Biology Examination Paper Types 2022

WAEC Biology OBJ Paper 1 :

The WAEC Biology paper one (OBJ) is the objective paper and it comes with 60 multi-choice OBJ questions. Candidates are to are answer these questions within 1 hour 30 minutes.

WAEC Biology Theory Paper 2 : WAEC Biology paper two is the Biology essay paper which is made up of different theory questions that you have to answer in detail.

Candidates will be given 2 hours to answer the WAEC Biology theory paper 2 questions. Each candidate is to answer about 5 to 6 questions in this section.

Get also: WAEC Timetable for 2022 PDF

WAEC Biology Objective (OBJ) Questions and Answers 2022 Expo

The following are some practice WAEC biology objective (OBJ) questions and answers 2022 expo to test you, which may possibly come out in the 2022 WAEC biology exam.

1. Which of the following human body systems is also known as the cardiovascular system?

  • Circulatory system
  • Endocrine system
  • Lymphatic system
  • Muscular system

The correct answer is the Circulatory system (Option A)

2. Which of the following is a blood group?

The correct answer is the AB (Option C)

3. Which of these Genotypes may possibly give birth to SS?

The correct answer is AS + AC (option D)

4. Each cell in a human contains ____ pairs of chromosomes?

The correct answer is 23 (option B)

Get also : free WAEC Chemistry questions and answers here

5. Human immune system fights off infection mostly through the use of ___?

  • Microbes and Antigens
  • Exogenous and immunogenic
  • Endogenous and Microbes
  • Antibodies and macrophages
  • Autoantigens and intracellular

The correct answer is Antibodies and macrophages (option D)

6. Which of these is not true about the human cells?

  • There are more bacteria cells in our body than our own human cells?
  • Most bacteria cells are essential to human health
  • Bacteria cells help us to perform key bodily functions
  • There are more non-human cells in our body than human ones
  • All bacteria cells in the human body are harmless

The correct answer is Option E

7. Which of the following is a function of the Appendix in the Human body?

  • It houses beneficial gut flora that can repopulate the digestive system whenever there’s a wipeout of the normal populations of this flora
  • It helps in building antigens in the body that can be transported to the human immune system to fight off infection
  • It stores essential minerals that can be converted into useful nutrients for the body.
  • It helps to store waste products that can be passed to the urinary tract to send out in the form of urine.

The correct answer is option A

8. Which of these Genotypes can marry without giving birth to SS?

The correct answer is AA + SS (option A)

9. Inhibitors are used to______?

  • Disrupt enzyme activity
  • Lower rate of reaction
  • Regulate an enzyme activity
  • Increase the rate of reaction

The correct answer is to Disrupt enzyme activity (option A)

10. What are the excretory organs in insects?

  • Stratum Corneum
  • Stratum Lucidum
  • Malpighian tubules

The correct answer is Malpighian tubules (option D)

WAEC Biology Theory (Essay) Questions and Answers 2022 Expo

The following are some samples and practice WAEC biology theory (essay) questions and answers to test your knowledge of biology.

1. Define metamorphosis.

Ans. Can be defined as the change of form or structure from an immature form to an adult form in two or more distinct stages.

2. State 5 harmful effects of parasites on their hosts

Ans. 5 harmful effects of parasites on their hosts are;

  • Parasites’ activities on their host can result in loss of weight in the host
  • Parasites can cause infections with bacteria in their host
  • Parasites can stimulate the production of toxic compounds that may alter the body function of their host
  • Parasites can cause harmful effects like immune suppression and make their host susceptible to other diseases
  • Parasites can cause allergic reactions in their host

3. State any 5 layers of the epidermis in the skin of humans

5, among the layers of the epidermis in the skin of humans, include;

  • The Basal Cell Layer
  • The Squamous Cell Layer
  • The Stratum Corneum
  • The Papillary Layer
  • The Reticular Layer

4. Explain why a Rhesus-negative woman who gets married to a Rhesus-positive man might possibly lose her second pregnancy.

Ans. A Rhesus-negative woman who gets married to a Rhesus-positive man might possibly lose her second pregnancy because a Rhesus-negative mother has no antigen on the surface of her Red Blood Cells/RBC while a Rhesus positive man has antigen on his RBC. So during her first pregnancy, the child carries a positive Rhesus factor at the pregnancy, the Rhesus factor enters the mother’s bloodstream and ruptures the placenta, and as a result of the rupture, blood seeps into the mother’s blood and causes it to develop anti-Rhesus antibodies which will then cause agglutination in a small proportion of foetal blood cells. So in the second pregnancy, the child might be Rhesus positive and a child carrying Rhesus positive will likely suffer massive destruction of red blood cells/erythroblastosis fetalis and that may result in a miscarriage of the child.

WAEC Biology Examination Expo 2022 Secrets

In this section, we are going to give you the WAEC expo for biology that the West African Examination Council has released to help candidates.

This expo is the corrections to the mistakes that candidates made during the biology exam and what you should do in your case.

The following are the WAEC expo tips on how to pass the WAEC biology exam 2022.

  • Make sure you answer all questions precisely without giving ambiguous answers
  • Make sure you supply your answers in the spaces provided
  • Show proficiency in answering questions that require recall of facts
  • Make sure you follow the examination instructions stated on the question paper
  • Make good expression in answering questions that require explanation
  • Learn how to and the rules in drawing and labeling of biology diagrams
  • Make sure you give titles to all the diagrams you draw (very important)
  • Learn how to spell technical terms in biology correctly

We believe this free WAEC Biology questions and answers 2022/2023, WAEC biology expo 2022, WAEC biology runs for biology paper 1 and paper 2, biology theory, and OBJ papers 2022 will be helpful to you.

Good luck in your exam!

guest

which part of the brain is responsible for memory storage

Nicholas

Yes I want to join

Blessing Godwin

Diseases in animals

Emmanuel Kwofie

What is DNA

Adekoya Mojolaoluwa

Pls send me everything

Nwifuru christopher

What aboutime practicals

Abdul Aziz

Waec Biology theory questions and answers 2022/2023

Sholanke

What is red blood cell

Oyinkansola

I want to join this group

Awolu Butnap dogo

Essay and objectives answer

Uthman RUKAYAT AYO

Metamorphosis

Jibrin nurudeen

Everything about it

Roland Davies

All theory and obj answers

Kolma Cletus

Thank u,the poster

John happiness

Effect of Kidney and remedies

Abdulazeez shakeerah

Biology waec answers obj and theory

Goodness

is this for today?

Jude Idris

Not. There are likely questions and answers

chukwumaemmanuep

Question n answer

Demson kangoma

I need biology essay questions paper 2022 wassce

Ope

How is the question going to look like

Not far from these questions

Abasienyene James

Abeg this wan na for tomorrow exam??🤨

Cyril Ogbomeda

I want to join

Taiwo

Essay is enough for me

Oluwah Tunmise

Biology question and answer both essay and objective

Jenny

BIology essy

Mary Samson Samson

Please I need 2022biology expo

Daisy Albert

Real biology question

Abdulganiyu Abdulqodir

Biology objective all answer

Edeh Ebube pricilia

Tissues and supporting system

Light

Tnks very much

ANONYMOUS

Pls I need the biology answers plsss

Ogono grandey

Please is this the current biology question

Okunlola feranmi

Show us all of biology question and answer for 2022

Ayomide

I need answers of WAEC Biology 2022/2023

Chimex

Mwalimu Resources

Hassle-free teaching and learning.

GENERATE YOUR OWN SCHEMES OF WORK. FROM PRIMARY (CBC) TO SECONDARY (8-4-4) IS SIMPLE I FASTER I ACCURATE

  • Premium Member Services
  • Pre-Primary
  • Primary School Holiday Assignments
  • Past KCPE Papers & Marking Schemes
  • 2022 KCPE Predictions
  • Grade 6 Term 2 Schemes
  • 2022 Grade 6 Schemes
  • CBC Lesson Plans
  • Grade 1 Exams
  • Grade 2 Exams
  • Grade 3 Exams
  • Grade 4 Exams
  • Grade 5 Exams
  • Grade 6 Term 1 2022 Exams
  • KPSEA Samples
  • Standard 7 Exams
  • 2024 Term 2 Opener Exam
  • 2024 T1 End Term Set 2
  • 2024 Term 1 End Term Set 1
  • 2024 Term 1 Mid Term Exam
  • 2024 Term 1 Opener Exam Set 2
  • 2024 Tern 1 Opener Set 1
  • 2023 Term 3 End Term Set 2
  • 2023 Term 3 End Term Set 1
  • 2023 Term 3 Opener set II
  • 2023 Term 3 Opener
  • 2023 Term 2 End Term Exam
  • 2023 Term 2 Mid Term Set 3 Exam
  • 2023 Term 2 Mid Term Set 2 Exam
  • 2023 Term 2 Mid Term Exam
  • 2023 Term 2 Opener Set 2 Exam
  • 2023 Term 2 Opener
  • 2023 Term 1 End Term Set 2 Exams
  • 2023 Term 1 End Term Exam
  • 2023 Term 1 Mid Term Set 2
  • 2023 Term 1 Mid Term
  • 2023 Term 1 Opener Examination
  • Term 2 2022 End Term Exams
  • Term 3 Opener Exams
  • Form 1 Term 3 Mid Term 2022
  • Form 2 Term 3 Mid Term 2022
  • Form 3 Term 3 Mid Term 2022
  • Form 2 Term 3 End Term 2022
  • Form 1 Term 3 End Term 2022
  • Form 3 Term 3 End Term 2022
  • Set II Term 3 2022 End Term Exam
  • KCSE Revision Booklets
  • 2024 Schemes of Work
  • 2024 April Assignments
  • 2023 December Holiday Assignments
  • 2023 April Holiday Assignments
  • December 2022 Holiday Assignments
  • Maswali na Majibu ya Chozi la Heri
  • Maswali ya Ushairi
  • Geography Revision Questions
  • Biology P1 Exams
  • Chemistry Practical Sets
  • Secondary Record of Work Covered
  • Secondary Lesson Plans
  • Teaching Vacancies
  • Nguu za Jadi
  • Chemistry Notes
  • Kiswahili Resources
  • English and Literature Resources
  • Business Studies Resources
  • A doll’s House – Character and characterization
  • Chozi la Heri-jalada
  • Maudhui- Chozi la Heri
  • History & Government Notes
  • Geography Notes
  • CRE & IRE Resources
  • How hormones influence growth and development in plants
  • How biotic factors regulate the population of animals in an ecosystem
  • How small intestines are adopted to their function
  • How are leaves of mesophytes suited to their function
  • Agriculture Topical Revision
  • Business Studies Topical
  • Biology Topical Revision
  • Chemistry Topical Revision
  • Physics Topical Revision
  • Introduction to CRE
  • Moses and Sinai Covenant
  • Loyalty to God: Elijah
  • History & Government Topical Revision
  • Earth and Solar system
  • Introduction to Agriculture
  • Factors influencing Agriculture
  • Crop Production 1
  • The early man
  • Entrepreneurship
  • 2024 LANJET PREMOCK
  • 2024 CHANGAMWE-JOMVU JOINT PREMOCK
  • 2024 BOKAKE Cluster Joint Pre-mock
  • 2024 Matete Cluster Pre-Mock
  • 2024 Kabarak High School Trail 1
  • 2024 Mang’u High T1 Pre-Mock
  • 2024 KCSE Pre Mock Set 2
  • 2024 Term 1 Pre-Mock Set 1
  • 2023 Kapsabet Boys Post Mock
  • 2023 Maseno School Mock
  • 2023 Asumbi Girls Mock
  • 2023 Peak Joint Mock
  • 2023 KALA Joint Mock
  • 2023 Samia Sub-county Cluster
  • 2023 Maranda Mock II
  • 2023 Lanjet Mock
  • 2023 Nyandarua County Trial Mock 4
  • 2023 Nyandarua County Trial 3
  • 2023 Achievers Joint Mock
  • 2023 CEKENAS Joint Mock
  • 2023 Kapsabet Boys Mock II
  • 2023 MOKASA II Joint Mock
  • 2023 BOKAKE Cluster mock
  • 2023 Mang’u High School Mock
  • 2023 BSJE Mock
  • 2023 Eagle KCSE Trial
  • 2023 LAINNAKU JOINT MOCK
  • 2023 NJARAKU Cluster Mock
  • 2023 MBORANU Joint Mock
  • 2023 KASSU JET
  • 2023 Murang’a Extra County Schools (MECS) Joint
  • 2023 ACK Mumias Diocese Mock
  • 2023 BUNAMFAN Pre- Mock Joint
  • 2023 Kakamega North Cluster
  • 2023 Kakamega Catholic Diocese Mock
  • 2023 Lugari Constituency Joint Mock
  • 2023 MOKASA 1 MOCK
  • 2023 Mumias West Joint Pre-Mock
  • 2023 SUKELLEMO Joint Pre-Mock
  • 2023 Sunrise Pre-mock
  • 2023 MECS Pre-Mock
  • 2023 Maranda High School Pre-Mock
  • 2023 Chogoria-Murugi Joint Pre-Mock
  • 2023 MOMALICHE 2 CYCLE 10
  • 2023 Arise and Shine Pre-Mock
  • 2023 Tiriki West Cluster
  • 2023 Kapsabet Boys Trial 1
  • 2023 Asumbi Girls F4 Entry Mock
  • 2023 Form 4 Term Mid Term Mock
  • Mokasa September 2022 Joint
  • Nyabururu, Homabay Kaplong, Rapogi Joint
  • CaSPA Eldoret Diocese Joint
  • Pangani Girls Nov 2022 Mock
  • Hexa Joint Evaluation
  • Nairobi School Sep 2022 Mock
  • Alliance High 2022 School Mock
  • Maseno School September 2022 Mock
  • Bondo Sub County Joint Exam (BSJE)
  • SUKELLEMO September 2022 Joint Mock
  • Kisii School September 2022 Mock
  • Mangú High School 2022 Mock
  • Londiani 2022 Joint Mock
  • Mathioya 2022 Joint Mock
  • MINCKS Group of Schools Joint
  • Mumias ACK Diocese Joint Mock
  • Sunrise September 2022 Mock
  • Catholic Diocese Kakamega Joint
  • Maranda September 2022 Mock
  • MECS Joint Sep 2022 Mock
  • CEKENAS September Mock
  • Kapsabet Boys September 2022 Mock
  • Asumbi Girls September Mock
  • Lanjet August September 2022 Mock
  • MOKASA Joint Mock 2022
  • Arise and Shine Mock 2022
  • Baringo North Mock Aug-Sep 2022 Mock
  • Maranda Pre-Mock 2022
  • KASSU JET JUNE 2022
  • MOMALICHE Joint- June 2022
  • Mumias West Mock-June 2022
  • BUNAMFAN Pre–Mock – 2022
  • CEKENAS Mock 2022
  • SUKELLEMO Pre Mock 2022
  • Kapsabet boys 2022 TRIAL 1 MAY
  • ASUMBI GIRLS HIGH SCHOOL PRE-MOCK
  • MANGU OCT/NOV 2021 MOCK
  • 2021/2022 KCSE Mocks
  • 2019 KCSE Mocks
  • KCPE Mocks Set 001
  • KCPE Mocks Set 002
  • KCPE Mocks Set 003
  • KCPE Mocks Set 004
  • 2023 KCSE Papers
  • 2022 KCSE Papers & Marking Schemes
  • 2021 KCSE Papers & Marking Schemes
  • 2020 KCSE Papers & Marking Schemes
  • 2019 KCSE Papers & Marking Schemes
  • 2018 KCSE Past Papers & Marking Schemes
  • 2017 KCSE Past Papers & Marking Schemes
  • CPA Resources
  • 2024 JSS Schemes
  • JSS Curriculum Designs
  • JSS Teaching Notes
  • 2024 Grade 7 Examinations
  • 2024 Grade 8 Examinations

BENEFITS OF PREMIUM MEMBERSHIP

  • Access to more resources (From primary to tertiary)
  • Access premium mocks
  • No advertisements
  • Bulky download of premium mocks (Can download all papers at once)
  • First access to new resources than other members
  • Request resources to be sent to your WhatsApp

Biology Essay Questions and Answers

Explain the factors that affect enzyme activity

Temperature; enzymes are protein in nature; and hence sensitive to temperature changes; as temperature increases, enzyme activity also increases until optimum/maximum; above this optimum the reaction decreases sharply; due to the destruction …

Biology Essays 

1. explain the various ways in which a typical cell is adapted to its functions.

Has a cell membrane; with pores; that regulates substances entering and leaving the cell; cytoplasm; contain sugars and salts; for maintaining its osmotic pressure; also has a liquid medium; for all biochemical reactions; nucleus; contain chromosomes having hereditary material; and controls all the activities of the cell; ribosomes; are sites for protein synthesis; golgi bodies/apparatus; for secretion of hormones and enzymes; formation of lysosomes; lysosomes; contain lytic enzymes for breaking down worn-out organelles; secretory vesicles; formed from golgi apparatus for secreting substances; smooth endoplasmic reticulum; synthesizes and transports lipids; rough endoplasmic reticulum; transport proteins; nucleolus; controls the activities of the nucleus; produces ribosomes; mitochondria; form sites for energy production; centrioles; formation of cilia and flagella; forms spindle fibres used in cell division; plant sap vacuoles; store salts and other dissolved substances; controls osmotic pressure and turgidity of cells; food vacuoles involved in digestion of engulfed food; chloroplasts; form sites for photosynthesis in plant cells; Max. 20 mks

2. Explain how the various specialized cells are modified to carry out their functions in plants and animals

Animal cells: Sperm cell; has acrosome containing lytic enzymes; that digest the egg membranes for penetration during fertilization; has a long tail; containing numerous mitochondria; to generate maximum energy for propulsion/swimming in the vaginal fluid after ejaculation; Red blood cells; are flattened, circular/spherical biconcave in shape; to increase the surface area for packaging of haemoglobin; has haemoglobin; that combines with respiratory gases; for transport to and from body tissues; White blood cells; are amoeboid in shape hence able to change shape; to engulf pathogens through phagocytosis; lymphocytes produce antibodies to fight pathogens; Nerve cell; has extensions/dentrites; to receive and send information for sensation; Ciliated epithelial cells; have cilia for propulsion of mucus that traps dust and micro-organisms in the respiratory tract; Muscle cells; elongated, striated and contractile; to bring about movement; Plant cells: Guard cells; bean-shaped; to regulate the size of the stomata allowing gaseous exchange; and control water loss; has chloroplasts with chlorophyll; for photosynthesis; Root hair cell; elongated; thin-walled; with dense cytoplasm for absorption of water and mineral salts; Epidermal cell; thin; for protection of inner tissues from mechanical and micro-organism attack; Palisade cell; contains numerous chloroplasts with chlorophyll; for photosynthesis; elongated; to increase surface area for trapping maximum amounts of light energy; Meristematic cell; thin-walled; with dense cytoplasm; for primary and secondary growth; Max. 20 mks

3. Describe how the mammalian body protects itself against infections

Pathogenic microbes are found on the skin, respiratory tract, mouth, vagina and the intestinal tract; the skin; has a keratinised and waterproof cornified outer layer; that provides a mechanical barrier to microbes/prevents entry of microbes; sebaceous gland; produces sebum; which has antiseptic properties; the respiratory tract; produce mucus secretions that trap dust; cilia sweep/waft/propel the microbes to the pharynx for swallowing or to be coughed out; reflex actions of coughing/sneezing/vomiting help remove foreign materials from the respiratory tract/digestive tract; lysozymes/enzymes in saliva/nasal secretions/tears; digest walls of bacteria destroying them; gastric secretions such as hydrochloric acid lowers the pH in the stomach killing micro-organisms; clotting of blood; prevents entry of microbes after damage of blood vessels; phagocytosis; by phagocytes engulf and destroy microbes and other foreign bodi es; lymphocytes are stimulated to produce antibodies; by proteins present in microbes protecting the body; antibodies destroy/kill micro-organisms through various ways: agglutinins; bind to pathogens making them clump together; killing them; Lysins; bind to pathogens and make them burst or disintegrate; opsonins; bind to pathogens making them easily recognized hence be engulfed/destroyed by other lymphocytes; anti-toxins; bind and neutralize toxins produced by micro-organisms; vagina is acidic; hence making it not conducive for growth and reproduction of micro-organisms; Max: 20 mks

4. How are the leaves of higher plants adapted to their functions?

Broad and flattened lamina; to increase surface area; for absorption of light; thin blade; to reduce distance for diffusion of gases and penetration of light waves; transparent epidermis and cuticle; to allow light to penetrate to tissues; cuticle layer absent on stomata; to allow for gaseous exchange; one-cell thick epidermal layer; to reduce the distance over which sunlight penetrates; palisade cells have numerous chloroplasts containing chlorophyll; to trap maximum amounts of light energy; have stomata on the epidermis; to allow for gaseous exchange; and control of water loss through transpiration; palisade layer have elongated cells located at right angles to the leaf surface; for maximum absorption of light energy; spongy mesophyll; consists of spherical and loosely-packed cells; to create air spaces; which communicate with the atmosphere through stomata; for purposes of gaseous exchange and control of water loss; veins have conducting tissues: xylem; for movement of water and dissolved mineral salts; phloem; for translocation of manufactured food; Max. 20 mks

5. Explain how the various teeth adapt mammals for nutrition

Incisor; sharp; chisel-shaped; for biting; and cutting food; one root for support in the jaw bone; Canines; long; sharp; pointed; for holding prey; piercing; and tearing flesh from prey; single root; for support in the jaw bone; Premolars; large/wide; to increase surface area for grinding food; highly cusped; to increase surface area for grinding food; two roots; for firm support/anchorage in the jaw bone; molars; large/wide; to increase surface area for grinding food; highly cusped; to increase surface area for grinding food; Max. 20 mks

6. Describe what happens to a meal of oily beans and maize from the time of ingestion up to the time of absorption

In the mouth; starch in maize; is digested by salivary amylase/ptyalin/diastase into maltose; food is chewed and mixed by teeth and the tongue; rolled into boluses by peristalsis; it enters into the stomach via the cardiac sphincter; in the stomach, gastric juice containing pepsinogen that is activated to pepsin; digests proteins in the beans; into shorter peptides; food is churned and allowed into the duodenum; via the pyloric sphincter muscle; in the duodenum; bile juice secreted by the gall bladder; emulsifies the oils in the beans into tiny oil droplets; pancreatic juice; secreted by the pancrease; contains pancreatic amylase; that digests starch to maltose; pancreatic lipase; that digests the oil in the beans to fatty acids and glycerol; trypsin; digests proteins into shorter peptides; food enters into the ileum; where succus entericus is secreted; it contains maltase enzyme; that digests the maltose into glucose; that is absorbed; peptidase; digests peptides into amino acids; lipase digests the remaining lipids (oil) into fatty acids and glycerol; which is absorbed through the lacteals of the villi; Max. 20 mks

7. How are the small intestines in mammals adapted to their functions?

Small intestines consists of the duodenum and the ileum; most digestion of food occurs in the duodenum; bile from the gall bladder of the liver is secreted through the bile ducts; and it is used to emulsify fats/break fat particles into tiny droplets; to increase the surface area for enzyme action; the pancreas secretes pancreatic juice to the duodenum; the juice contains pancreatic amylase; that helps to breakdown the remaining starch into maltose; trypsin; (that is secreted in its inactive form, trypsinogen, and activated by enterokinase enzyme); hydrolyses proteins into shorter peptides; pancreatic lipase; converts lipids into fatty acids and glycerol; sodium hydrogen carbonate is also produced; to neutralize the acidic chyme from the stomach; and provide a suitable alkaline medium for pancreatic and other intestinal enzymes; the ileum is long; and narrow; to increase the surface area for complete digestion of food; and maximum absorption of digested food; highly-coiled; to reduce speed of food flow; for maximum digestion; and absorption; presence of villi; and microvilli; to increase surface area; for maximum absorption; dense network of capillaries; to transport blood; for efficient transport of absorbed food; presence of lacteals in the villi; for absorption of fatty acids and glycerol molecules; presence of enzymes: Lipase; for digestion of lipids into fatty acids and glycerol; maltase; for digestion of maltose to glucose molecules; peptidase; for breakdown of peptides into amino acids; sucrase; for digestion of sucrose into glucose and fructose; lactase; for digestion of lactose into glucose and galactose; goblet cells; produce mucus; to lubricate the walls of the ileum; for smooth flow of food; coats the walls of ileum to prevent digestion by peptidase enzyme; Max. 20 mks

8. Outline and explain the various homeostatic functions of the liver in mammals

Deamination; process of removal of an amino group from an amino acid molecule; the process gets rid of excess amino acids in the body; as the body is not able to store them; the amino group enters the ornithine cycle; where it is combined with carbon (IV) oxide to form urea; which is excreted in urine through the kidney; Heat production; many metabolic activities take place in the liver; releasing heat energy; that is distributed by the blood to other parts of the body; this helps in thermoregulation; Storage of vitamins and mineral salts; Vitamins A, B, D, E and K; are stored in the liver; worn-out red blood cells, are broken down to yield iron; which is stored in the liver in form of ferritin; this is used later in case of shortage; Formation of red blood cells; occurs in the liver of the foetus; the liver also breaks down old/exhausted red blood cells; leading to formation of more in the bone marrow to replace the worn-out cells; to enhance oxygen and carbon (IV) oxide distribution; Regulation of blood sugar level; liver cells convert excess glucose into glycogen and fats under the influence of insulin hormone; the stored glycogen is however converted back to glucose; when glucose levels are low; by the liver cells; under the influence of glucagon hormone; Regulation of plasma proteins; plasma proteins such as prothrombin and fibrinogen are manufactured in the liver using the amino acids found in the liver; they play a major role in blood clotting; that prevents excessive blood loss and infection at the injured area; other plasma proteins produced by the liver such as serum and albumen; contribute to the maintenance of osmotic pressure in the body; non-essential amino acids are also synthesized by the liver; for use by the body; Storage of blood; the liver is highly vascularised; hence it is capable of holding a large volume of blood when the blood vessels dilate during hot conditions; when the temperatures are low, the blood vessels constrict under the influence of the endocrine and nervous systems; hence less blood is stored in the liver; this contributes to thermoregulation; Detoxification; this is the process where harmful compounds such as drugs and poisons; are converted to less toxic compounds in the liver; toxicity is caused by medication, drugs and microorganisms; the toxic compounds are later excreted in urine; detoxification prevents the accumulation of toxins in body cells; which could lead to death or malfunctioning of the body cells; Max. 20 mks

9. Explain why the following conditions are necessary for photosynthesis

A) carbon (iv) oxide.

Required in the dark stage of photosynthesis; it combines with the hydrogen ion from the light stage; to form glucose, proteins and lipids; low concentrations reduce the rate of production of energy and food; while high concentrations leads to an increase in the amount of energy and food formed;

It is used to break down water molecules (through photolysis); into hydrogen ions, oxygen and energy; the energy and hydrogen ions formed are used in the dark stage;

c) Chlorophyll 

  • Green pigment that traps light energy from the sun; that is used in photolysis of water molecules;

d) Suitable temperature and pH 

  • Temperature affects the enzymes involved in photosynthesis; suitable/optimum temperatures activate enzymes; for maximum production of food; while extremely low temperatures inactivate enzymes; leading to less or no production of food; high temperatures denature enzymes; stopping the process of photosynthesis; photosynthetic enzymes work well in low pH; so the rate is high; while higher pH reduces enzyme activity; lowering the rate of photosynthesis; ;

Forms a medium for the chemical reactions; it is split to yield hydrogen ions, oxygen and energy for use in the dark stage; solvent for the materials used in photosynthesis; Max. 20 mks

11) Explain how endotherms respond to heat and cold conditions in their environment

Heat/hot conditions: Increased sweating; to lose heat through latent heat of vaporization; dilation of arterioles under the skin; to bring more blood to the skin surface to lose heat to the atmosphere; decreased body metabolism; to reduce heat generation; erector pili muscles relax; making hair follicles to relax hence hair lies flat on skin, no air is trapped; to lose heat; slow/reduced muscular activity due to slow metabolism; to reduce heat production; panting to expose tongue and mouth; to release heat; moving to shades to avoid direct heat; aestivation; to escape the extreme heat; flapping of ears to create currents to carry away heat; Cold conditions: stamping of feet; to generate heat; basking in the sun to gain heat directly; less production of sweat; to reduce water loss through latent heat of vaporization; vasoconstriction of arterioles; hence less blood flow to the skin surface to reduce heat loss; increased metabolism through release of more thyroxine hormone; to generate heat; erector pili muscles contract; pulling hair follicles hence hair is raised; to trap a layer of moist air; to prevent heat loss; shivering/rapid contraction of muscles; to yield heat to warm body; Max. 15 mks

12) Describe the route taken by water from the soil up to the evaporating surface of a plant

Water is drawn into the root hair cells by osmosis; due to the presence of dissolved substances in the cell sap of root hairs, the concentration of cell sap is greater than that of the surrounding solution in the soil/concentration gradient; this exerts a higher osmotic pressure, thus drawing the water molecules across the cell wall and cell membrane into the root hair cells; more water drawn into the root hair cells dilutes the cell sap; making it less concentrated than that in the adjacent cortex cell of the root; due to osmotic gradient, water moves from the adjacent cells to the next by osmosis; until it enters the xylem vessels located in the center of the root; the xylem vessels of the root then conduct the water up into the xylem vessels in the stem into the leaves; there is a force in the roots which pushes water up the stem; this force is known as root pressure; and can be considerably high in some plants; energy from the endodermal cells of the root is responsible for driving this force; in the xylem vessels, water would rise up by capillarity; to some extent because the vessels are narrower and there is a high attractive force between the water molecules and the cell walls; the cohesive; and adhesive forces are important in the maintenance of a continuous and uninterrupted water column in the xylem vessels up the tree to the leaves; water vaporizes from the spongy mesophyll cells; their cell sap becomes concentrated than the adjacent cells. This increases the osmotic pressure of the spongy mesophyll cells; as a result of this, water flows into the cell from other surrounding cell, which in turn takes in water from xylem vessels within the leaf veins; this creates a pull/suction force that pulls a stream of water from xylem vessels in the stem and roots. This force, known as transpiration pull; helps in maintaining a continuous column of water from the roots to the leaves; water flows from the midrib into leaf veins from where it enters leaf cells; from the mesophyll cells, it enters the airspaces; then the substomatal air chambers; from where it evaporates through the stomata; to the atmosphere; Max. 20 mks

13) How is the mammalian heart adapted to its functions

Heart is enclosed in a pericardial membrane/pericardium; that produces a fluid; to lubricate it; the membrane also keeps the heart in position; It is covered in a fatty layer; that acts as a shock absorber; made up of cardiac muscles; which are interconnected/interacted hence contract and relax without fatigue or nervous stimulation/myogenic; for continuous pumping of blood throughout the lifespan of the animal; the muscles are supplied by nutrients and oxygen; by the coronary arteries; and the coronary veins take away wastes and carbon (IV) oxide; heart is divided into 4 chambers; for efficient double circulation/ avoid mixing of oxygenated and deoxygenated blood/carry large volume of blood; has interventricular septum; to separate oxygenated and deoxygenated blood; ventricles are thick/muscular; to generate high pressure to pump blood out of the heart; left ventricle has thick muscles/more muscular; to pump blood to all body tissues; heart has bicuspid; and tricuspid valves; to prevent back flow of blood to left auricle; and right auricle respectively; valves have tendinous cords/valve tendons; to prevent them from turning inside out; semi lunar valves located at the beginning of major arteries; prevent backflow of blood into the ventricles; has sino-artrio node located in the muscles of the right auricle; to initiate heart beat/contractions of heart muscles/cardiac muscles, rate of heart beat is controlled by nerves; vagus nerve; slows down heartbeat; while sympathetic nerve; speeds up the heartbeat; has aorta; to transport oxygenated blood to all body parts; has pulmonary artery; that transports deoxygenated blood from right ventricles to lungs for oxygenation; has pulmonary vein; that transports oxygenated blood from lungs to the left ventricles; for distribution to all body parts; has the venacava; that receives deoxygenated blood from all body parts to right ventricles; Max. 20 mks

14. Describe double circulation in mammals

Deoxygenated blood from body tissues (except lungs); enters the heart via the right auricle; through the venacava; it flows to the right ventricle; via the tricuspid valve; the right ventricle contracts; pumping blood; via the semi lunar valves; through the pulmonary artery; to the lungs for oxygenation; the oxygenated blood from the lungs; flow through the pulmonary vein; to the left auricle; via the bicuspid valve; to the left ventricle; the left ventricle contracts; pumping blood via the semi lunar valves; through the aorta; to the rest of the body tissues; Max. 20 mks

15. Describe the process of urine formation in the mammalian kidneys

The afferent arteriole which is a branch of the renal artery supplies blood to the glomerulus; the afferent arteriole has a wider lumen/diameter than the efferent arteriole; which takes away blood from the glomerulus; the differences in the diameter of the afferent and the afferent vessels causes high pressure; leading to ultrafiltration of blood; the walls of the blood capillaries are one-cell thick; hence glucose, amino acids, vitamins, hormones, salts, creatine, urea and water filter into the Bowman’s capsule; to form glomerular filtrate; white blood cells, red blood cells, plasma proteins such as globulin and platelets are too large to pass through the capillary wall; hence remain in the blood capillaries; useful substances in the human body are selectively reabsorbed; back into the blood stream at the proximal convoluted tubule; the tubule is highly coiled; to increase the surface area for reabsorption of the substances; the useful substances include amino acids, glucose, vitamins, hormones, sodium chloride and water; many mitochondria found at the proximal convoluted tubule; provide energy for reabsorption of these substances against a concentration gradient; the glomerular filtrate flows into the descending and the ascending limb of the loop of Henle; blood in the capillaries and the glomerular filtrate in the loop of Henle move in opposite directions/counter-current flow; this provides a steep concentration gradient that leads to maximum absorption of water through osmosis; sodium chloride is actively absorbed from the ascending limb into the blood capillaries; under the influence of aldosterone hormone; the glomerular filtrate flows into the collecting tubule from where, more water is reabsorbed into the blood stream; antidiuretic hormone influences the amount of water to be reabsorbed depending on the osmotic pressure of the blood; the glomerular filtrate from several collecting tubules now referred to as urine; is emptied into the collecting duct; the urine passes through pyramid, pelvis and ureter into the bladder; where it is stored for some time. The sphincter on the urethra relaxes to allow urine to be released from the body; Max. 20 mks

16. Explain the role of the following hormones during homeostasis

A) antidiuretic hormone (adh).

Secreted by the (posterior lobe/end) pituitary gland; in response to an increase in the osmotic pressure of blood; the hormone stimulates the distal convoluted tubules and the collecting ducts; to increase their permeability to water; this increases the reabsorption of water into the bloodstream; concentrated and less urine is excreted; when the osmotic pressure decreases, less or no hormone is produced; hence the tubules become impermeable to water; less water is reabsorbed into the bloodstream; hence more dilute urine is excreted; fluctuations in the osmotic pressure is detected by the hypothalamus;

Secreted by the pancrease; in response to a rise in blood sugar level; it stimulates liver cells to convert the excess glucose into glycogen and fats for storage in the liver and muscle cells; increases the oxidation of glucose in respiration to yield water energy and carbon (IV) oxide/increases metabolism in the body; this leads to a fall in blood glucose to normal level;

c) Glucagon

Secreted by the pancrease; in response to a decline in blood glucose level; it stimulates liver cells to convert the stored glycogen and fats back to glucose; stimulates the conversion of amino acids to glucose; and stops the oxidation of glucose in the body cells; the glucose formed is released to the blood stream causing a rise of blood glucose level to normal; Max. 20 mks

17. a) Distinguish between Diabetes mellitus and Diabetes insipidus

Diabetes mellitus is a condition/disease caused by failure of the pancrease to produce adequate insulin hormone; leading to excess glucose levels in the body some of which is released in urine while diabetes insipidus is a condition caused by failure/inability of the kidney tubules to control the amount of water in urine as a result of a defect in production of antidiuretic hormone (ADH) leading to production of more dilute urine; Max. 2 mks

b) Explain how mammalian bodies regulate glucose and protein levels in their blood

When glucose level is high (above 90mg/100cm3), the brain sends impulses to the (β cells of islets of Langerhans) pancrease cells; to release insulin hormone; the hormone stimulates liver cells to convert the excess glucose into glycogen and fats for storage in the liver and muscle cells; increases the oxidation of glucose in respiration to yield water energy and carbon (IV) oxide/increases metabolism in the body; this leads to a fall in blood glucose to normal level; However, when the glucose level falls below normal (below 90mg/100cm3); the brain sends impulses to the (α cells of the islets of Langerhans) pancrease cells; which are stimulated to release glucagon hormone; the hormone stimulates liver cells to convert the stored glycogen and fats back to glucose; stimulates the conversion of amino acids to glucose; and stops the oxidation of glucose in the body cells to avail more glucose; the glucose formed is released to the bloodstream causing a rise of blood glucose level to normal; The level of plasma proteins such as prothrombin, globulins, albumins and fibrinogen; which play a major role in osmoregulation and blood clotting; are controlled by the liver; which manufactures them using the amino acids found in the liver; when their levels reduce, more is produced; but when the level is high, less of the proteins is produced in the liver; excess amino acids are deaminated; as the body is not able to store them; the process involves removal of an amino group from an amino acid molecule; the amino group enters the ornithine cycle; where it is combined with carbon (IV) oxide to form urea; which is excreted in urine through the kidneys; Max. 18 mks

18. Explain how the various abiotic factors may affect plants

Temperature; affects soil formation and distribution of plants; affect transpiration rate as high temperatures lead to high rates of transpiration; It also affects the rate of photosynthesis with the direct influence on enzyme activity; Light intensity; affects the rate of photosynthesis; Wind; increase the transpiration rates; affects dispersal of seeds and fruits; agents of pollination; affect distribution in terms of wind storms/breakages; Atmospheric pressure; high atmospheric pressure leads to low rates of transpiration; high oxygen and carbon (IV) oxide concentration; high photosynthetic rates; while low atmospheric pressure leads to high transpiration rates; less concentration of oxygen and carbon (IV) oxide; leading to low rates of photosynthesis; Water/Rainfall; forms a raw material for photosynthesis; helps in support in plant tissues; affects distribution of plants; Humidity; low humidity leads to high transpiration rates; while high humidity leads to low rates of transpiration; pH; affects distribution of plants; some grow in acidic soils; others in alkaline soils; Edaphic/soil factors; affects plant distribution; in terms of being sources of water and mineral salts; provide a substratum for anchorage of plants; Max. 20 mks

19. Discuss the causes, effects and control measures for water pollution

Causes of water pollution are varied: industrial effluents; have heavy metals that poison aquatic organisms; untreated organic matter has phosphates/sulphates/nitrates/salts; that cause eutrophication; causing algal bloom that deprives the water of nutrients; when the algae die, they lead to an increase in putrefying bacteria whose decomposition activities lead to the release of awful smells/odours; oil effluents clog respiratory surfaces of aquatic organisms/death due to suffocation; domestic effluents/sewage; form a habitat of pathogens that spread water borne diseases; decomposing sewage promotes eutrophication leading to algal bloom; death promotes/attracts saprophytic bacteria that use up oxygen in water; causing suffocation/death to aquatic organisms; agrochemicals/fertilizers; phosphates/nitrates; cause eutrophication; heavy metals in agrochemicals (herbicides/pesticides); affect respiratory surfaces/cause breathing problems; Hot water; raise temperature of water; killing organisms; dissolves less oxygen; reducing its content in water; Oil spillage; in oceans from tanks/refineries; soak feathers of marine birds preventing flight; clogs respiratory surfaces leading to death; coats photosynthetic phytoplanktons; reduces light penetration hampering photosynthesis; Sediments; from soil erosion makes water dirty; making it unfit for consumption; clogs respiratory surfaces hindering gaseous exchange; reduces light penetration hindering photosynthesis; Control methods: Enforcement of environmental laws; Use of unleaded fuel/petroleum products; Proper treatment and disposal of sewage wastes; Treatment of industrial effluents before release; Public education on correct use of inorganic fertilizers and agrochemicals; and use of alternatives such as biological control of weeds/pests/organic manure; Use of undersea pipelines instead of tankers to transport oil products; Cooling hot water before release to water bodies; Max. 20 mks

20. How are xerophytes and hydrophytes adapted to their habitats?

Xerophytes: thick waxy cuticle; minimize water loss; leaves are folded and reduced in size; to minimize stomatal transpiration; sunken stomata; to reduce rate of transpiration; thick/succulent leaves, side branches or stems; for water storage; shedding of leaves during the dry periods; to reduce surface area exposed for transpiration; reversed stomatal rhythm; prevent excessive loss of water; deep penetrating roots; to absorb water from deep below the surface; superficial roots; to absorb surface water run-off; leaves covered in scales/hairs; to trap a moist layer of air; to reduce the rate of transpiration; drought-resistant seeds; that remain dormant till favourable weather resumes; underground organs (corms/bulbs); for storage of water and reproduction; most stomata located on the lower leaf surface; to avoid exposure to direct light; to reduce evaporation; reduced number of stomata; to reduce the rate of transpiration; Hydrophytes: stomata on the upper surface of leaves; to provide a large surface area for gaseous exchange; and loss of excess water; poorly-developed roots that lack root hairs; to reduce/avoid absorption of water; aerenchyma tissue in leaves, stems and roots; to store air; and for buoyancy; deeply-dissected leaves; to provide a large surface area for absorption of light; highly-sensitive; and numerous chloroplasts; for photosynthesis; greatly-reduced vascular bundle; to avoid absorption of water; flowers raised above the water; to allow for pollination; lack of a cuticle or very thin cuticle; for faster loss of water; Max. 20 mks

21. Outline the differences between wind and insect pollinated flowers

Flowers of wind pollinated plants are small; with no bracts, sepals or petals; if present the petals are small, inconspicuous; often white or green in colour; while insect pollinated flowers are large; often with brightly coloured petals, bracts or inflorescence; to attract insects. Flowers of wind pollinated plants have no nectaries; and no scent; while flowers of insect pollinated plants are scented; and produce nectar; in wind pollinated flowers, the anthers are large; and loosely attached on a flexible filament; to allow pollen grains to be readily released when wind blows on the anthers; while anthers of insect pollinated flowers are usually small; and firmly attached on the filaments; this ensures that the insect rub against the anther; as they crawl into the flower collecting pollen grains onto their bodies; in wind pollinated flowers, the stigmas are feathery; widely spread; this acts as nets to catch pollen as it floats through the air; while in insect pollinated flowers the stigmas are small; smooth; and sticky; and are also enclosed; this feature ensures that pollen grains from the body of an insect stick onto it; in wind pollinated flowers, the flowers are simple with no particular shape; while some flowers that are insect pollinated have petals with grooves or dark lines; leading from the petal boarder to the nectaries; some have tubular or funnel-shaped corolla; and landing platforms; to guide the insect to the source of the nectar for their food; flowers of wind pollinated plants are either on long stalks above the leaves; or develop from flower buds that open before the leaf buds; to increase the flower exposure to air currents; while flowers of insect pollinated plants are on short stalks; often enclosed by the corolla; Max. 20 mks

22) Describe what happens in a flower from the time of pollination up to the time of seed and fruit development

After pollination, the pollen grain absorbs nutrients from the stigma; and develops a pollen tube; it grows down the style to the embryosac; taking along the male nuclei; the tube nuclei initiates and maintains pollen tube growth; while the generative nucleus divides by mitosis; to form two male gamete nuclei; which follow behind the tube nucleus as the pollen tube grows down the style; pollen tube enters the ovule through the micropyle; its tip bursts open; while the tube nucleus disintegrates; one of the male gamete nucleus fuses with the egg cell nucleus/oosphere/megaspore; to form the zygote; while the other fuses with the two polar nuclei; to form a triploid nucleus; called the primary endosperm nucleus; after fertilization, the zygote undergoes repeated mitotic divisions; to form an embryo consisting of the plumule, radicle and seed leaves/cotyledons; primary endosperm nucleus divide repeatedly, become separated by membranes; to form an (semi-fluid nutritive) endosperm; ovary walls change into the pericarp; ovary changes/develops into a fruit; while ovules lose water and become seeds; the integuments; change into seed coats/testa; style/filaments/petals/sepals wither and fall off (or may persist); Max. 20 mks

23. Discuss the adaptations of the female reproductive system of humans

Elastic uterine walls; to expand so as to accommodate the growing foetus; muscular foot of the pelvis and bladder support the weight of the growing foetus; funnel-shaped ends of the oviduct; direct the ova released to the uterus; muscular uterine walls; contract and relax to expel the foetus at birth; long vaginal canal; allow sufficient entry of penis to avoid wastage of sperms; the two ovaries maximize chances of releasing ovum after every circle (28 days); ovaries are well vascularised/have good blood supply; to ensure nourishment of cells involved in oogenesis (primordial mother/germ cells) or egg formation; high number of potential mother cells; ensures maximum number of ova which develop to maturity; plenty of yolk in egg cells; which nourish the foetus before the placenta becomes functional; the vitelline membrane of the ovum thickens after fertilization; preventing further entry of sperms; the oviduct wall is able to contract; in order to facilitate movement of ovum down the oviduct; has cilia to waft the ovum forward; wall of the vagina/vulva produce mucus; to lubricate the penis during copulation; clitoris; has many nerve endings; to provide maximum stimulation during copulation for maximum ejaculation and faster movement of spermatozoa; Max. 20 mks

25. How is the mammalian gaseous exchange system adapted to its functions?

Gaseous exchange occurs in the spongy mesophyll; During the day, air diffuses into large air spaces of the spongy mesophyll; through the stomata; the carbon (IV) oxide in the air diffuses into photosynthetic cells; in solution form; during photosynthesis, carbon (IV) oxide is used up; while oxygen is produced; some of the oxygen is used in respiration; while the rest diffuses out of the leaf; through the stomata; During the night, air diffuses into the air spaces; through the stomata; the air dissolves into the film of moisture; oxygen in the air diffuses into the cells; and is used for respiration; carbon (IV) oxide produced; diffuses out through stomata; due to a concentration gradient/diffusion gradient; At night, carbon (IV) oxide accumulates in the leaf since photosynthesis does not occur; some gaseous exchange also takes place through the cuticle; and through the epidermis of young leaves, roots and stems; some plants exchange gases through breathing roots/pneumatophores; older stems exchange gases through lenticels; Max 20 mks

24. Describe the process of gaseous exchange in terrestrial plants

Nasal cavity; has cells that produce mucus; that together with hairs/cilia; trap and propel dust/microbes to the pharynx to be breathed out/swallowed; cavity is supplied with capillaries; that warm the air for faster flow in the channels; epiglottis; covers the trachea during swallowing; so that particles of food and water may not enter the trachea; trachea and bronchi; have cartilage rings; to keep the passages open/prevent them from collapsing; so that air moves in and out freely and continuously; are also lined with mucous membranes which have hairs/ciliated; whose movement/wafting push out dust particles collected in the passages into the pharynx; richly-supplied with blood vessels; to warm the air; for faster flow; lungs; have numerous alveoli; to increase the surface area for gaseous exchange; alveoli have a thin epithelium; to reduce the distance through which gases diffuse for easier and faster diffusion; alveoli are moist; to dissolve oxygen for faster transport; lungs are spongy; because of many air sacs that contain a large amount of/volume of air; Lungs are also supplied with many blood vessels; for transportation of gases; they are also supplied with a network system of trachea, bronchi and bronchioles; to provide an efficient system/large surface area for gaseous exchange; Lungs are enclosed in a pleural membrane; which secrete pleural fluid; that protect the lung surface; lubricate the chest cavity; allowing smooth movement of lungs as they change volumes; ribs have intercostal muscles; that moves/contracts and relaxes to allow for inhalation and exhalation; ribs also protect the lungs; has the diaphragm muscles whose contraction and relaxation leads to inhalation and exhalation respectively; Max. 20 mks

26. Describe the structure and function of the mammalian skin

It has a cornified layer made up of dead cells and is tough and impermeable to water; to protect the skin against mechanical damage; bacterial infections and water loss; granular layer; whose cells divide to form the cornified layer; malpighian layer; which is made up of diving cells that give rise to a new granular layer; contains melanin; to protect skin against ultra-violet rays/radiations; Sebaceous glands; which secrete sebum; to make the skin supple/soft and waterproof; sebum is also antiseptic; Blood vessels; dilate during hot weather; increasing blood flow near the skin surface; heat loss is enhanced; constrict; in cold weather; less blood flow; minimize heat loss; Sensory nerve endings and receptors; enable detection of external environmental changes; Highly coiled sweat glands; secrete sweat; to control body temperature; when hot sweat evaporates cooling the body; sweat contains excretory products; subcutaneous fat/adipose tissue in dermis; for insulation; hair; to regulate body temperature; in cold weather erector pili muscles contract; hair is raised, air trapped to insulate the body; in hot weather, erector pili muscles relax; hair lies flat reducing insulation; dense network of blood capillaries; supply nutrients/oxygen to skin tissues; as well as carrying away wastes and carbon (IV) oxide away from the skin tissues; adipose tissue/sub-cutaneous layer; serves as an insulator; helping in temperature control; helps in manufacture of vitamin D; Max. 20 mks

27. Describe the role of the following hormones in the menstrual cycle

A) luteinising hormone (lh).

Produced by the pituitary gland; under the influence of oestrogen hormone; cause the bursting of the Graafian follicle; to release a mature egg/ovum/causes ovulation; stimulates the change/conversion of the Graafian follicle; into the corpus luteum; stimulates the corpus luteum; to secrete progesterone hormone;

b) Follicle Stimulating Hormone (FSH)

Produced by the anterior lobe of the pituitary gland; it stimulates the maturation of the Graafian follicle in the ovaries; stimulates the ovarian tissue/wall to secrete oestrogen;

c) Oestrogen

Brings about/stimulates the healing and repair of the uterine wall; after menstruation; stimulates the pituitary gland; to secrete luteinising hormone;

d) Progesterone

Secreted by the corpus luteum; it stimulates the thickening of the endometrium/uterine wall; in preparation for implantation; inhibits secretion of the Follicle Stimulating Hormone; therefore preventing further development of the Graafian follicle; Max. 20 mks

28) Describe the process of secondary thickening in a woody stem

Facilitated by meristematic cells (cambium) located between the phloem and the xylem (intervascular cambium); it divides radially to form cambium tissues; with xylem forming the outer ring/to the inside; while the phloem forming the outer ring/to the outside; division of the cambium ring; form a secondary parenchyma; hence increases/forms the medullary rays; other xylems (secondary xylem) are formed; hence pushes the phloem and cambium ring outwards; this creates pressure on the outer cells; resulting in stretching and eventual rupturing of epidermal cells; a new band/volume of cambium cells are formed in the cortex beneath the epidermis (cork cambium cells/phellogen); to replace these ruptured cells; the phellogen cells divide on either side; where the inner cells become the secondary cortex; while those produced on the outside become cork cells; which are tightly packed; and become coated with an oily/waxy water-proof material/suberin; further multiplication of cork cells; lead to formation of the bark; which forms a protective layer (against water loss and damage by organisms); seasons results into annual rings; some cork cells form a loose mass/lenticels that allow gaseous exchange through the stem; Max. 18 mks

29. Discuss the various mechanisms of opening and closing of stomata

Photosynthetic theory; during the day, guard cells carry out photosynthesis manufacturing glucose; This increases the osmotic pressure of the sap vacuole; which becomes higher than that of the neighbouring epidermal cells; guard cells therefore take in water by osmosis; and become turgid; the outer thin wall stretches easily; pulling the thicker inner wall outwards; thus the stomata opens; At night, there is no light hence no photosynthesis takes place; plant cells respire using up more glucose; the osmotic pressure of the sap vacuole of the guard cells reduces; becoming lower than the neighbouring epidermal cells; the guard cells lose water by osmosis; to adjacent epidermal cells; they then become flaccid; pulling together the thick inner walls; and stomata closes; Enzymatic inter-conversion between starch and glucose/sugar; At day time, plants continuously use carbon (IV) oxide for photosynthesis; leading to an increase in the pH of the guard cells; this causes starch to be converted to sugar/glucose; the glucose increases the osmotic pressure of the guard cells; hence water is taken in by osmosis; the cells become turgid and bulge outwards; causing the stomata to open; At night, no photosynthesis occurs but respiration takes place; carbon (IV) oxide accumulates in guard cells; lowering the pH; the low pH favours conversion of glucose into starch; starch is osmotically inactive; this lowers the osmotic pressure of guard cells; guard cells therefore lose water by osmosis to the adjacent epidermal cells; become flaccid; pulling together the thick inner walls; and the stomata closes; Active ion exudation; during the day, there’s an accumulation of potassium and sodium ions; as a result of active pumping of the ions by the ATP formed through photosynthesis; carbon (IV) oxide fixation occurs in the guard cells; the guard cells become turgid; and stomata open; At night, before the stomata close, the ions diffuse out of the guard cells into epidermal cells; the osmotic pressure of guard cells is lowered; they lose water to epidermal cells by osmosis; and become flaccid; thereby closing the stomata; Max. 20 mks

b) Discuss three examples of natural selection in action

Melanic forms of peppered moths; in Europe, there are two forms of peppered moths; white and black; before industrialization, the tree trunks were white; therefore the white peppered moths were white; hence were camouflaged; the black varieties were easily noticed and fed upon by predatory birds; the white form therefore reproduced and increased in number; during industrialization, the smoke from industries coated tree trunks black; the black variety became camouflaged; reproduced and increased in population; the white variety were easily noticed and fed upon by predators; they reduced in population; Resistance against drugs and antibiotics; where microorganisms are continually exposed to a certain drug; their cells synthesise specific proteins; which counter the drug; this ability to synthesize the protein is passed onto the offspring; Resistance to pesticides by insects; insects such as mosquitoes when continually exposed to a particular pesticide; synthesize a specific protein which make them resistant to the pesticide; this is then inherited by their offspring; Max. 18 mks 31. Discuss Lamarck’s and Darwin’s theories of evolution Larmarck’s theory states that when the environment demands the need or use of a particular structure in the body; the body develops it in response; for example giraffes used to have short necks; when all the grass was exhausted, they started stretching their necks in search of leaves on trees; therefore they developed long necks; which then were inherited by their offspring; however, when a structure is not continually used, it reduces in size and becomes dysfunctional; this theory fails to explain how acquired characteristics become inherited; Darwin’s theory suggests that in nature there occur struggle for existence; only those individuals with the desired adaptations survive; those poorly adapted fail to compete; and become extinct; there also occurs variation in nature; where organisms with desired adaptations pass on their characteristics to offspring during reproduction; those poorly adapted fail to reach maturity; and do not reproduce; therefore, nature selects for individuals best suited to an environment; and against those poorly adapted (natural selection); as there occurs survival of the fittest; Max. 20 mks

32. a) Explain how the following blood cells are adapted to their functions

i) Red Blood Cell

Presence of haemoglobin molecules; with a high affinity to combine with oxygen as/to form oxyhaemoglobin; bi-concave shape; to increase the surface area for packaging of haemoglobin; absence of nucleus; to accommodate maximum/more haemoglobin molecules; thin membrane; for faster diffusion of gases; Max. 6 mks

ii) White blood cell

Irregular in shape/amoeboid; which changes to enable the cell to squeeze through the capillaries; lymphocytes produce antibodies; which help to prevent diseases; phagocytes are amoeboid-shaped; to change shape and engulf bacteria cells; Max. 6 mks 

  b)Explain the different ways in which Carbon (IV) Oxide is transported by blood

Carbon (IV) oxide diffuses out of the tissues into the red blood cells where it reacts with water; in the presence of carbonic anhydrase enzyme; to produce carbonic acid; The acid dissociates into hydrogen and hydrogen carbonate ions; the hydrogen carbonate ions then diffuse out of the red blood cells into the plasma; where it further dissociates to produce carbon (IV) oxide on reaching the alveolar cavities of the lungs and diffuses into the alveoli; some carbon (IV) oxide combines with the amine group in the haemoglobin molecule forming carbaminohaemoglobin; which dissociates in the lungs producing carbon (IV) oxide; some carbon (IV) oxide dissolves in the blood plasma forming carbonic acid, which dissociates to carbon (IV) oxide on reaching the lungs; Max. 8 mks

33. Describe how the following vertebrae are adapted to their functions

Has a wide neural canal; to accommodate the large spinal cord at the neck region; has large/broad wing-like cervical ribs; to increase the surface area for attachment of the neck muscles; has facets on the anterior side; for articulation with the occipital condyles of the skull; allowing up and down movement/nodding of the head; has posterior facets for articulation with the anterior facets of the axis; forming a joint that allows sideways movement of the head;

Has a broad centrum; that projects to form the odontoid process; for articulation with the neural canal of the atlas; a joint that allows turning of the head; has a large and broad/flattened neural spine; and flat cervical ribs; to increase the surface area for attachment of neck muscles;

Has many transverse processes; and additional projections (metapophyses, hypapophyses, anapophyses); to offer a large surface area for attachment of abdominal muscles; broad neural canal; to allow passage of the large spinal cord at the upper abdominal area; large/thick centrum; to support the weight of the body; and withstand strains/upthrust force due to movement;

d) Thoracic

Long/elongated neural spine; to offer a large surface area for attachment of the large back muscles; have a large centrum and neural canal; to offer support to the thoracic cage; has tubercular facet on the transverse processes; to articulate with the tuberculum of the ribs; while the capitular demifacets on the centrum; articulates with the capitula of the ribs; together with the ribs and the sternum form the thoracic/rib cage; for protection of heart and lungs; and for breathing process; Max. 20 mks

34. a) Why is locomotion necessary in higher animals?

Animals move in order to look for food; mates; escape danger/predators; look for shelter/ suitable environmental conditions; 4 mks

b) Explain how bony fish are adapted to their habitats

Have streamlined bodies; to reduce friction; body is covered with scales; which overlap backwards; to reduce friction; skin produces mucus; which covers the body making it slippery; reducing friction; have swim bladder; which stores air; for buoyancy hence make the fish float; myotomes/muscle blocks; that contract alternately; for forward thrust in water; lateral line system; on either side of the body which is sensitive to pressure and water currents; possess fins; that are used for locomotion: tail/caudal fins; for propulsion; dorsal; and anal fins; prevent rolling; pectoral fin; used for breaking/steering; prevents yawing/side to side movement; controls pitching; pelvic fins; for steering/pitching; Max. 16 mks

35. Describe how the various supportive tissues in plants adapt them to their habitats

Sclerenchyma tissue; long, slender cells with tapering ends; with walls thickened with lignin; provide support and protection to the more delicate tissues; and resistance to storms and strong winds; main constituent of wood; Xylem vessels; longitudinally-elongated cells; with perforated end walls; with heavily lignified walls; to increase rigidity and strength to the plant; a main constituent of wood; Tracheids; mainly found in angiosperms; made up of long tapering dead cells; cell walls are highly lignified; and pitted; cells lie in large overlapping groups; to offer extra support; Collenchyma tissue; longitudinally elongated living cells; located beneath the epidermis and mid rib of leaf veins; thickened at the corners by cellulose and pectin compounds; to provide support in leaves, herbaceous plants and young woody plants; Parenchyma tissue; large; spherical cells; with thin cellulose walls; forming the bulk of cortex and pith of most plants; become tightly packed and rigid when turgid; to attain and maintain an erect posture of plants; main support structures in herbaceous stems/plants; Max. 20 mks

36. a) What is a reflex action?

Rapid and automatic; response to a particular stimulus; 2 mks

b) Outline the activities that occur in the body when one touches a hot object

When the hot object is touched, the pain receptors; in the skin of the finger are stimulated; nerve impulses are initiated and transmitted through the sensory neurone; to the grey matter; of the spinal cord to the brain; for interpretation; the impulses are then transmitted through the relay neurone; via a synapse; the impulses from the relay neurone are transmitted via the motor neurone; through another synapse; to the effector; which are the biceps muscles of the upper arm; making the muscles to contract; straightening the arm; and the arm is withdrawn from the hot object; Max. 18 mks

37. Describe the nitrogen cycle

This is the cycling of nitrogen and its compounds in nature; plants absorb nitrogen in form of nitrates and then assimilate it into plant proteins; animals obtain this nitrogen in plant proteins through feeding on plants; when the animals die and decompose, they release the nitrogen in form of ammonia to the soil; free atmospheric nitrogen is converted into nitrates through a process known as nitrogen fixation; the process occurs in two ways: biological and non-biological; biological fixation of nitrogen is done by nitrogen-fixing bacteria; which are either free-living or symbiotic; symbiotic bacteria are of the genus Rhizobium; and are found in root nodules of legumes (such as pea, clover and alfalfa); the bacteria convert atmospheric nitrogen into ammonia; that is used directly by the leguminous plants to form nitrogen containing organic compounds (amino acids, nucleic acids, proteins); when plants die, the nodules release ammonium compounds into the soil; which are then converted to nitrites; by nitrifying bacteria of genus Nitrosomonas and Nitrococcus (nitrite bacteria) and then to nitrates by Nitrobacter (nitrate) bacteria; free-living micro-organisms that fix nitrogen include putrefying/saprophytic bacteria; (such as Azobacter spp, Clostridium and some algae such as Anabaena, Chlorella and Nostoc); the organisms fix nitrogen into ammonia by break down of protein material in dead organisms; the ammonia is converted to nitrites; then to nitrates; However, denitrifying bacteria (e.g. Pseudomonas denitrificans and Thiobacillus denitrificans); break down/reduce nitrates to nitrites, ammonium compounds and even gaseous nitrogen; a process known as denitrification; the process helps to release free nitrogen into the air for recycling; non-biological nitrogen fixing is carried out by lightning during thunderstorms; the lightning energy, causes atmospheric nitrogen and oxygen to combine forming oxides of nitrogen; which dissolve in rain water to form nitrous acid/nitric acid; that is washed down into the soil; the nitric acid formed reacts with other chemical compounds dissolved in soil water; to form nitrates; the nitrates are then utilized by plants; Max. 20 mks

38. Discuss how the various tropisms adapt plants to their habitats

Phototropism; growth curvature in response to direction of light; enables plant shoots to grow and get light for maximum photosynthesis; allows for leaf mosaic; Thigmotropism; growth curvature in response to contact/hard surface; makes plants with weak stems to get support on large plant/trees; this makes them to reach and get light for maximum photosynthesis; Geotropism; growth curvature in response to gravity; enables plant roots to grow deep into the soil for maximum support/anchorage; Hydrotropism; growth curvature in response to moisture/water; water is then used as a raw material during photolysis stage of photosynthesis; Chemotropism; growth curvature in response to chemical concentration gradient; enables pollen tubes to grow down the style and into the ovary for fertilization to occur in plant flowers; Thermotropism; growth curvature in response to temperature changes; enables plants to grow to where they can acquire optimum temperature for effective plant process (e.g. sunflower orientates towards the direction of the sun); Max. 20 mks

39. Discuss the various evidences of organic evolution

Comparative anatomy/taxonomy; members of a phylum/group show similarities; organs have similar structure/organs performing the same function such as the digestive system, urinary system, vertebrate heart; homologous structures are structures with the same embryonic origin but have been modified to perform different and specific functions; show a form of divergent evolution; e.g. the pentadactyl limb in vertebrates which has been modified for racing; swimming and flight or beaks of finches and birds; while analogous structures are those with different embryonic origin but have been modified to perform the same function e.g. wings of insects, bats and birds; eyes in octopuses and humans; show a form of convergent evolution; vestigial structures; have been reduced in size and become functionless; in the course of evolution; e.g. limbs in snakes, human hair and tail; Cell biology/cytology; occurrence of similar organelles such as the mitochondria and the endoplasmic reticula point to common ancestry; Fossil records/Paleontology; remains of organisms preserved in naturally-occurring materials for many years; fossil records show morphological changes of organisms over a long period of time e.g. skull of humans and horse; they provide a direct evidence of existence of organisms at a particular ecological era; however, since only hard parts are preserved, no evidence is available for existence of soft-bodied organisms; and there are many missing links; since remains are accidentally preserved in rudimentary rocks and resins; Comparative embryology; vertebrate embryos are morphologically similar during the early stages of development; suggesting that the organisms had a common ancestry/origin e.g. larvae of mollusks/annelids, embryos of chicken, humans, sheep; the closer the semblance between embryos, the closer their ancestral backgrounds; Geographical distribution; present continents are thought to have been a large land mass joined together; as a result of continental drift; isolation occurred bringing about different patterns of evolution; where plants and animals from different continents yet with common ancestry can no longer interbreed; because they evolved into different species; examples of animals that moved to different areas are the jaguars and Llamas in south America, lions in Africa, Tigers in Asia, marsupials in Australia; Comparative serology/physiology; semblance in blood components such as blood proteins, antigen-antibody reactions, structure of haemoglobin in all vertebrates; reveal some phylogenic relationship among organisms/show common ancestry; Max. 20 mks

40. Describe the structure and functions of the various parts of the mammalian ear

Pinna; is wide/funnel-shaped to collect/gather sound waves; and direct them to the auditory canal into the ear; Eardrum/tympanic membrane is thin and light; to convert sound waves into vibrations; Ear ossicles/maleus, incus and stapes are of high density; to magnify/amplify sound waves; Oval window is smaller than eardrum; to magnify the sound waves; and direct them to the inner ear; Cochlea is long and coiled; to increase surface area; for attachment of receptor cells/sensory hairs; cochlea has many sensory hairs; which receive sound vibrations and generate impulses; Liquid or fluid/endolymph in cochlea; transmit sound vibrations; auditory nerve; transmit impulses to the brain for interpretation; Eustachian tube; link the mouth and middle ear to equalise pressure; between middle and outer ear to prevent damage to delicate eardrum; Round window; lose excess vibrations; to avoid continuous stimulation; Semi-circular canals; contain receptors for body balance and posture; External auditory canal cells produce/secrete wax; to trap dust particles/solid/micro-organisms that can damage eardrum; Max. 20 mks

41. Discuss the various ways employed by preys to avoid the predators

Some preys resemble inedible inanimate and animate objects; this is called mimicry; e.g. walking stick insect resembles dry twigs of plants, some moths look like bees or flowers of some plants; this prevents birds from easily notifying and eating them; many have the ability to run very fast; because of having muscular bodies; and long legs; enabling them to escape predators e.g. antelopes, zebras; some have a body colour that resembles the surrounding; which helps them to camouflage or conceal in the background environment; e.g. zebras, giraffes; some graze in large herds; this enables them to fight off predators; e.g. wildebeests and buffaloes; some have evolved tough skin or coverings like shells; which can not be broken by some predators e.g. snails, tortoises, armadillo; production of foul smell e.g. in skunks; that discourages the predators; confrontational display that can scare away the predator e.g. porcupine; large eyes on both sides of the head give animals such as zebra a wide field of vision; enabling them to keep track of their enemies from far; and take precautions; Max. 20 mks

42. a) What is meant by the term symbiosis?

Nutritional association of two different organisms (2 plants or between an animal and plant); for mutual benefit; the relationship enables the composite organism to survive where neither can live on its own; Max. 2 mks

b) Describe five types of symbiotic relationships in a natural ecosystem

Lichens; these are composite plants consisting of blue-green algae; within a mycelial mass of a fungus; algal cells are provided with support, obtain water, carbon (IV) oxide and minerals and protection from fungus; while the fungus obtains oxygen and the carbohydrates made by algae; this enables the plants to survive on hard bare rocks in high attitudes and polar regions; Leguminous plants and nitrogen-fixing bacteria; the bacteria multiply and fix nitrogen from air into nitrates for the benefit of the plant; bacteria are protected and obtain nutrients from the plants; Ruminants and bacteria; the rumen has bacteria that secrete cellulose; that digests cellulose in the food/vegetation consumed by the animal to glucose for the animal; while the bacteria get shelter and use part of digested food; Mycorrhizal fungi and higher plants; the fungi found on forest trees gain photosynthetic organic products made by the trees; while the trees get nutrients/minerals absorbed by the fungus from the soil; Tryconympha and termites; the former is a protozoan living in gut of termites; and produce cellulase enzyme; that digests cellulose from the plant into digestible products for the benefit of the termite; the termite on the other hand provides shelter and protection; and absorbs some of the food for its use; Max. 18 mks

43. a) Describe the adaptations of Schistosoma spp to their parasitic mode of life

The parasite utilizes two hosts; the snail and humans; to increase chances of transfer of the parasite from one place to another; have suckers for attachment to host walls; to prevent them from being dislodged; the parasite produces many larval forms (e.g. miracidia, cercariae and redia) in snails; to increase chances of transmission and survival; as this feature poses barriers/difficulties in efforts aimed at eradicating the parasite; cercariae larvae and eggs of the parasite have glands that secrete lytic enzymes; which soften the tissues of humans/snails; to allow for penetration; chemical substances produced by the adult worm; protects the parasite from the action of the hosts’ defense mechanisms; they exist as separate sexes; with the male carrying the female; this ensures that eggs produced by the female are fertilized before being shed into the blood stream; Max. 15 mks

b) Outline five measures that can be employed to prevent and control the spread of the parasite

Proper disposal of human waste; urine and faecal material should not be disposed in water bodies to avoid contamination by the eggs or adult worms; drainage of stagnant water pools and use of molluscides to kill the intermediate hosts (snails); avoid swimming/bathing in snail-infested water bodies; wearing protective clothing such as gloves and gumboots when working or walking in swampy areas; personal hygiene that includes washing hands after visiting the toilet and drinking of boiled or chemically treated water to kill the eggs and the larval forms in the water; proper treatment of infected persons; Max. 5 mks

44. Describe the process of mitosis

Occurs in somatic/body cells; through five main stages/phases: Interphase/Resting stage; intense internal activities occur in the cell at this stage in preparation for the division; the activities include; replication of each chromosome to multiply genetic material to retain chromosomal number in daughter cells; chromosomes appear as a diffuse tangle of threads (chromatin); synthesis of new cellular organelles; build-up of energy stores (ATP) to drive the entire cell division process; Prophase; chromosomes become visible; as they shorten and thicken appearing as discrete strands (chromatids) lying parallel to each other; in animal cells, centrioles separate and move to opposite ends (poles) of the cell; they radiate from each of the ends forming spindle fibres; nuclear membrane begins to breakdown; nucleolus disappear; Metaphase; chromosomes migrate/move to the centre of the cell; and align themselves along the equatorial plane of the spindle; they get attached to the chromosomes, by their centromeres; nuclear membrane breaks down and disappears; spindle fibres lengthen; and attach to the centrioles at both poles forming asters; Anaphase; chromatids separate at the centromere; shortening of the spindle fibres occurs; resulting in the chromatids migrating to opposite poles of the cell; spindle apparatus begins to disappear; Telophase; final stage where chromatids reach the poles; become densely packed together and uncoil; a nuclear membrane forms around each mass/set of chromatids (now referred to as chromosomes); cytoplasm divides into two (cytokinesis); in animal cells, the cytoplasm divides by constriction of the cell membrane; while in plant cells, a cell plate forms within the cytoplasm and grows to separate the cell into two; spindle fibres disappear within the cytoplasm; and nucleoli reappear in the nuclei; of the two daughter cells formed at the end of telophase; Max. 20 mks

45. Discuss the various mechanisms that hinder self-pollination and self-fertilisation in plants

Protandry and protogyny; these are mechanisms where either the male or female parts of the reproductive organs ripen at different times in some flowers; Protandry is a case where stamens ripen earlier; and anthers release their pollen grains before the stigma is mature; while protogyny refers to a case where the stigma matures earlier; and hence becomes ready to receive pollen grains before the anthers are ready/ripe to shed the pollen grains; common in plants of the grass family; Self-sterility or incompatibility; is a case where pollen grains cannot germinate on stigma of the same plant; but only germinate on a different plant of the same species; hindering self-pollination; Heterostyly; condition of having different arrangements of style and stigma; for instance flowers could have shorter stamens than pistils; hence becomes impossible for the pollen to land, germinate and fertilise the ovules of the same flower; pistils on some flowers could also be shorter than the stamens therefore other mechanisms that hinder self-pollination are utilized; Dioecius and monoecius plants; dioecius plants have reproductive parts located separately on different plants of the same species; discouraging self-pollination; while monoecius plants have the parts located at different parts of the same plant body; encouraging cross-pollination; Max. 20 mks

46. How are seeds and fruits of plants adapted to their mode of dispersal?

Water; Fruit mesocarp/seed testa has air spaces; thus light/buoyant to float; carried away by water; fruits/seeds protected from soaking by waterproof pericarp/testa; Animal; have hooks for attachment to animals; thus carried to other places; fruits are brightly coloured; succulent/fleshy; aromatic/scented; to attract animals; which feed on them; the seed coats/hard seeds are resistant to digestive enzymes; thus are unaffected; seeds dropped away from parent in faeces/droppings; Wind; have hairs/wing-like structures/floss/extensions; which increase surface area/for buoyancy; making it easy to be blown away; fruits/seeds are light due to small size; therefore easily carried away by wind; censor mechanism; perforated/open/split/capsule; usually loosely attached to the stalk/long stalk; is swayed by wind; scattering seeds; Self-dispersal/Explosive mechanism; tension/pressure is created inside a dry pod; pod opens (violently) along lines of weaknesses; the two halves curl outwards; scattering the seeds; Max. 20 mks

 47) Describe the causes and effects of chromosomal mutations

Deletion; refers to the absence of a portion of a chromosome; it results from breakage and falling off of a portion of a chromosome; leading to loss of a group of genes that may have a disastrous effect on the development of an organism; Inversion; refers to reversal of normal sequence of genes in portion of a chromosome; occurs when a middle portion of a chromosome breaks, turns or rotates (inverts) through 180o and joins up again; this does not change the genetic constitution of the organism; but may bring into close proximity genes whose combined effects to an organism produce a beneficial effect to an organism; or cause disadvantages to the organism; Translocation; attachment of a portion of a chromosome to a non-homologous chromosome; occurs when a chromosome breaks and the portion joins another non-homologous chromosome; this may lead to serious consequences, even death depending on what genes are missing; Duplication; situation where a set of genes is represented twice in a chromosome; a part of a chromatid formed during cell division may replicate further to form an extra piece; which may attach onto the same or another chromatid; resulting to traits controlled by some genes being excessively expressed; Non-disjunction; this is failure of a pair of homologous chromosomes to separate during the first stage of meiosis; resulting in one of the daughter cells formed after division of the cell having two of one kind of a chromosome; while the other cell has less or none; diseases or disorders known as syndromes are known to result from this aberration e.g. Down’s Syndrome (Mongolism),Turner’s Syndrome, Klinefelter’s Syndrome; Polyploidy; this is the presence of more than two sets of chromosomes in a cell; occurs due to a failure of a cell to divide after the first stage of meiosis or after the chromosomes have replicated in mitosis; common in plants than animals; in plants, it causes some improvements such as resistance to drought, certain diseases and pests, improved yields and early maturity; Max. 20 mks

48. a) What is transpiration?

Process by which plants lose water to the atmosphere; in form of vapour (through lenticels, stomata and cuticle); 2 mks

b) Explain how the various environmental factors affect the rate of transpiration

Temperature; an increase in temperature increase the water vapour holding capacity of air in the spaces between mesophyll cells due to increased evaporation; therefore more water diffuses from the cells increasing the water vapour pressure; this causes an increase in the diffusion gradient between the intercellular spaces and the atmosphere; hence increasing the rate of transpiration; low temperature decreases water vapour pressure and the diffusion gradient hence; lowering the rate of transpiration; Light intensity; high light intensity increases the rate of photosynthesis in the guard cells; causing the opening of stomata; leading to increased water loss; it also increases the internal temperature of the leaf; that increases the evaporation rate in the intercellular spaces; leading to a higher rate of transpiration; low light intensity; reduces the rate of water loss; Humidity; a humid atmosphere lowers the water vapour diffusion gradient; hence lowering the rate of water loss; in a less humid/dry atmosphere, water diffusion gradient is high/steep; hence the rate of transpiration increases; Availability of water in the soil; more water will diffuse to the atmosphere when there is adequate or excess water; as more will be absorbed; increasing the rate of water loss; the guard cell will also remain turgid; hence stomata are open; leading to more water loss; however, less water in the soil leads to a reduced diffusion gradient between the mesophyll cells and the atmosphere; thereby reducing the rate of transpiration; Wind/air currents; wind blowing over a leaf surface carries pockets of moisture away from the leaf; creating a steep diffusion gradient between the atmosphere and the leaf; increasing the rate of water loss; in still air/on a calm day however, water vapour at the leaf area becomes saturated; and the diffusion gradient is lowered; reducing the rate of transpiration; Atmospheric pressure; low atmospheric pressure leads to a high rate of diffusion of water vapour; since air molecules move at a faster rate; and this increases the rate of water loss; in high atmospheric pressure conditions however, there is low rate of diffusion of water vapour; hence the rate of water loss is lowered; Max. 20 mks

50. Discuss the role of the various hormones in plant growth and development

Indole Acetic Acid/Auxins; influences/promotes cell division/elongation (in cambium causing secondary thickening); induces tropisms; promotes fruit formation/parthenocarpy; promotes formation of abscission layer/leaf fall; promotes cell differentiation (of vascular tissue); causes apical dominance/inhibits lateral bud formation; promotes growth of adventitious roots; in conjunction with cytokinins, it induces callous tissue formation; Gibberellins/Gibberellic acid; promotes cell division/elongation in dwarf plants; promotes parthenocarpy; setting of fruit after fertilization initiating formation of fruits; formation of side branches of stems/ends apical bud dormancy; inhibits adventitious root growth; activates hydrolytic enzymes in germination/promotes germination of seeds/breaks seed dormancy; affects leaf expansion and shape/retards leaf abscission; Cytokinins (e.g. Zeatin or Kinetin); promotes flowering in some plant species; breaks dormancy in some plant species; promotes cell division in presence of auxins; stabilizes protein and chlorophyll; promotes root formation on a shoot; low concentration encourages leaf senescence/increases cell enlargement in leaves; stimulates lateral bud formation; Ethylene/Ethene; promotes ripening of fruits; induces thickening of stem/inhibits stem elongation; promotes flower morphogenesis/formation or flowering in pineapples; causes abscission of leaves/fruit/leaf fall; Abscissic acid; high concentration causes stomatal closure; inhibits stem elongation/growth; inhibits sprouting of buds/induces bud dormancy; inhibits seed germination/growth/causes/promotes seed dormancy; causes abscission of leaves/fruits/leaf fall; Traumatin; heals wounds by promoting callous formation; Florigen; promotes flowering; Max. 20 mks

51. Explain how the process of evolution may result to the formation of a new species

For a new species to be formed, a population of organisms must become completely isolated or separated from others; over long periods of time so that any new variations that arise will not therefore flow to the other population; there are various isolation mechanisms: Geographical isolation; this is due to physical barriers such as oceans/seas/deserts; Ecological isolation; a barrier resulting from the occupation of different types of habitats from the original type; it may be due to isolation for reasons of feeding/predation/breeding; as well as environmental changes such as climate and vegetation; which may result in a population living in different habitats; to become ecologically separated from one another; Behavioural isolation; alteration of behaviour proceeding mating; which include courtship behaviour/lack of attraction between males and females in different populations; due to production of different chemicals or pheromones or colouration/songs; Reproduction isolation; a barrier to successful mating between individuals of a population; due to structural differences in reproductive organs; as well as failure in fertilization/incompatibility; Genetic isolation; even if fertilization takes place; the zygote may be inferior/fails to develop; however if the zygote develops, the offspring may be inferior or infertile/sterile; Max. 20 mks

52.) Discuss the various ways in which anaerobic respiration is utilized in industries and homes

Bread making; yeast is used to ferment sugar in wheat flour into carbon (IV) oxide and energy; the carbon (IV) oxide is produced in form of bubbles that causes the dough to rise and become porous; Beer making; yeast is used to ferment sugars in malt/grapes/fruits; to form beer, wines and spirits; Sewage treatment; anaerobes break down raw sewage and harmful industrial effluents; to harmless products of water, energy/heat and carbon (IV) oxide; Silage formation; vegetation is fermented by bacteria to produce nutritious and good-scented/flavoured animal feed that increases production/yields; Production of acids and strong liquors; special bacteria and fungi ferment food products; to produce acids such as citric acid, oxalic acid and vinegar; the products are used as food preservatives and flavouring agents; Manufacture of dairy products; under controlled environments; anaerobes help in fermentation hence manufacture of milk products such as butter, cheese, ghee and yorghurt; Production of fuels such as biogas; and gasohol; cane sugar is fermented by yeast; to produce gasohol for running engines or operating machinery; animal wastes such as guano and cow dung; can be used to produce a mixture of methane and carbon (IV) oxide gas; by exposing it to fermentation agents; methane is used to run simple machines such as water pumps and for cooking; Production of fermented porridge and milk; maize or wheat flour and milk is exposed to microbes in the air which ferment it; to produce sour and sweet tasting porridge or milk; Max. 20 mks

53. a) What is digestion?

Digestion is (mechanical and chemical) the process by which large complex food molecules; are broken down into soluble molecules (for absorption across intestinal wall to bloodstream); 2 mks

b) What is the importance of a balance diet in human nutrition?

A balanced diet consists of all the food types in their right proportions; it includes proteins, carbohydrates, minerals, lipids, water, vitamins and roughage; Proteins; used for growth and repair of worn out tissues; provide energy incase of acute shortage of carbohydrates/starvation; bind and transport specific molecules from one part of the body to another; structural proteins support tissues in the body e.g. bone and cartilage tissues; act as metabolic regulators such as enzymes and hormones; Carbohydrates; used to produce energy/store energy; Lipids produce energy; form of storage of energy; insulate the body; major structural components of the cell membrane; when oxidised, it provides metabolic water; Vitamins; defense against infections; form coenzymes which activate enzymes; Water; a universal solvent; medium for chemical reactions; used as coolant because of its high specific heat capacity; maintains the shape of cells; hydrolysis of many substances; transport medium in the body; Mineral salts; formation of teeth and bones; formation of hormones (e.g. thyroxine); formation of blood; maintenance of osmotic pressure of body fluid; transmission of nerve impulses; Roughage; makes food to be bulk; promotes peristalsis; and absorption of water in the large intestines; induces mucus production; Max. 18 mks

NECO Biology Questions and Answers 2021/2022 [Theory & OBJ Expo]

  • April 5, 2022

Updated NECO Biology Questions and Answers 2021: This is an online Solution to NECO Biology Questions 2021 and sample answers expos for Theory & OBJ papers.

This solution is made available so as to help NECO candidates looking for NECO Biology Questions and Answers 2021 expo solution for paper 1 and paper 2.

NECO Biology Answer paper Sheets

Download NECO 2021 Timetable PDF Here

I must also let you know that this NECO 2021 Biology Questions and Answers expo solution is free for all NECO candidates

  • Register NECO Online

NECO Biology OBJ & Theory Questions and Answers 2021

Have you ever wondered and thought of how you can possibly write this NECO Biology exam and come out with a very good grade?

If you say yes, then this is the very best NECO Biology Questions and Answers 2021 solution material that is highly recommendable to all NECO Candidates.

RECOMMENDED: Download NECO Timetable PDF

Please, I will advise candidates to take their time and follow this solution material step by step because it will really them to pass this NECO Biology exam right now.

NECO Biology Exam Guide: once you get to your exam venue, make sure you locate the exam Hall.

Once you get the exam paper, make sure you pray.

After praying to God, read through the exam instructions.

Now, make sure you fill in your candidate’s exam details which include exam Number, full name, Center number, etc.

NECO Biology OBJ Questions and Answers Expo 2021

Below are some sample NECO 2021 Biology questions and answers to start with. Note, they are just sample questions provided to give you insight on the kind of questions you should be expecting.

Take note of them, check their correct options, and keep them in memory in case you see any in the exam. We may post the real and correct answers for 2021 anytime. So keep checking this page.

1. Which of these is a mixed gland?

Correct Answer is Pancreas

2. ______ are Tissue which cells have lost the capacity of cell division?

  • Meristematic tissue
  • Permanent tissue
  • Both Mesristmatic and Permanent tissue
  • None of these

The Correct Answer is Permanent tissue

3. Why do wooden doors get difficult to open or close during Rainy Season? because of;

  • Plasmolysis

Correct Answer is Imbibition

4. Agrostology can be define as the study of__________?

  • Seed of oil

Correct Answer is Grass

5. How many pairs of Peripheral nerves are in man________?

 Correct Answer is 43

6. What is found in eukaryotic cells that are not prokaryotic?

Correct Answer is A) Nucleus

7. The total salt content in the blood is around_____?

  • 85 to 11.9%

Correct Answer is : D) 0.85 to 0.9%

8. Which of these hormones accelerate the rate of a heartbeat?

  • acetylcholine

Correct Answer is A) adrenaline

9. The human heart is also Known as….?

  • Neurogenic heart
  • Myogenic heart
  • Pulsating heart
  • Ampullary heart

Correct Answer is: B) Myogenic heart

10. The father of Zoology is__________?

  • Theophrastus

 The Correct Answer is: B) Aristotle

11. Spermology is the study of_________?

The Correct Answer is: B) Seed

12. Which of this doesn’t have a red blood cell?

The Correct Answer is: B) Earthworm

13. Which of the following does Chambered heart occur in______?

Correct Answer is: D) Cockroach

14.The Amount of energy released by 1 gram of glucose is equal to to_____?

Correct Answer is: B) 4 kcal

15. Why is the corner of the cells of a collenchyma tissue in the plant thickened?

Because of the deposition of;

  • Lignin and suberin
  • Suberin and cutin
  • Cellulose and pectin
  • Chitin and lignin

Correct Answer is C) Cellulose and pectin

NECO Biology Theory Answers to Questions 2021

Below are awesome tips and instructions for the the NECO Biology theory questions and answers 2021.

Please kindly follow the instructions below to answer any NECO biology Theory question, then follow these steps:

  • Read and understand the instructions on the paper.
  • Make sure you read a question twice before answering.
  • Make sure you understand what the question is about before answering.
  • Let your answer be very short, simple and explanatory.

RECOMMENDED: GET MORE NECO UPDATES HERE

Download Free 2021 NECO Syllabus for all Subjects below. Select each and download NECO Biology Syllabus here

Please kindly use this NECO Biology Questions and answers 2021 as a hint and instruction to answering every other questions.

biology essay questions and answers 2022

15 comments

Is it the true answer

I need the theory

Ok we will update the Theory soon

Pls is the questions right

I need NECO biology OBJ and theory

I need assistance for neco science subjects

I need theory and objectives

Is It The Real Question For 2021 Neco

Those are possible questions

I need the neco theory

Leave a Reply Cancel reply

Your email address will not be published. Required fields are marked *

Save my name, email, and website in this browser for the next time I comment.

ATBU Cut Off Mark 2021/2022 (Departmental & JAMB CutOff Point)

6 best shower caddies for college student dorms, you may also like, waec gce mathematics questions, answers 2020/2021 & syllabus.

  • April 26, 2022

WAEC Syllabus for Government 2021/2022 & Textbooks : Download PDF

  • April 25, 2022

WAEC English Questions and Answers 2022/2023 |Oral, Essay & Theory

  • February 8, 2022

WAEC GCE Commerce Questions, Answers 2020/2021 & Syllabus

  • April 19, 2022

2022 NECO Physics (OBJ & Essay) Answers [29th July, 2021]

  • March 29, 2022

NECO Mathematics Questions and Answers 2022/2023 | OBJ & Theory

  • April 7, 2022

IMAGES

  1. Biology questions and answers

    biology essay questions and answers 2022

  2. AQA Combined Science Biology Paper 1 Sample Questions and Model Answers

    biology essay questions and answers 2022

  3. GCSE Biology: Multiple-Choice Topic Question Pack On Inheritance

    biology essay questions and answers 2022

  4. A-level biology essay on the importance of cycles in biology

    biology essay questions and answers 2022

  5. WASSCE Biology Questions For 2022 Final Year Students

    biology essay questions and answers 2022

  6. NECO Biology Questions And Answers 2022

    biology essay questions and answers 2022

VIDEO

  1. Biology Paper 2 Que & Ans KCSE 2022|KCSE Paper 2| Biology Revision KCSE 2022

  2. Final advice, tips and strategy for paper 3

  3. 2023 AL

  4. 12th Complete Biology MVVI Questions for Board 2024

  5. 11th Biology 10 Important Long Questions 2024

  6. 2023 Biology paper 2 questions and answers part 2

COMMENTS

  1. PDF Biology Essay Questions And Answers

    KCSE BIOLOGY BIOLOGY ESSAY QUESTIONS WITH MARK SCHEMES 1. Explain the various ways in which a typical cell is adapted to its functions Has a cell membrane; with pores; that regulates substances entering and leaving the cell; cytoplasm; contain sugars and salts; for maintaining its osmotic pressure; also has a liquid

  2. KCSE 2023 Biology Essay Questions and Answers (KCSE 2023 ...

    Here are KCSES 2022/2023 Biology Essay Questions and Answers (KCSE 2022 Prediction Questions). Content: 31 pages with 60 questions and answers. BIOLOGY (231/2) Revision Questions (Essays): Expected Responses. Download Essay Qns. Q1. Explain the various ways in which a typical cell is adapted to its functions.

  3. Biology KCSE Essay Questions and Answers Paper 2; Over 1,000

    Biology Simplified Notes Form 1 to 4 Free. Biology Free notes and Exams for Form one to Four. BIOLOGY KCSE MARKING AND SETTING TIPS IN LINE WITH EMERGING TRENDS. Biology topical questions and answers. Biology KCSE Past Papers and all marking schemes free downloads. Biology free lesson plans for all topics (Form one to four)

  4. PDF AP Biology 2022 Free-Response Questions

    Directions: Questions 1 and 2 are long free-response questions that require about 25 minutes each to answer. Questions 3 through 6 are short free-response questions that require about 10 minutes each to answer. Read each question carefully and completely. Answers must be written out in paragraph form.

  5. AP Biology Past Exam Questions

    Free-Response Questions. Download free-response questions from past exams along with scoring guidelines, sample responses from exam takers, and scoring distributions. If you are using assistive technology and need help accessing these PDFs in another format, contact Services for Students with Disabilities at 212-713-8333 or by email at ssd@info ...

  6. AQA A Level Biology

    The importance of shapes fitting together in cells and organisms. 1) Enzyme properties and digestion. 2) Protein structure. 3) Plasma membrane structure and cell transport. 4) Antigens, antibodies, B cells & T cells. 5) Vaccines. 6) Structure of DNA. 7) DNA Replication (not PCR) 8) Transcription & translation.

  7. AP Bio

    The AP Biology exam used to consist of eight long-form free response questions, but in 2019 it was redesigned to consist of only six free response questions. Two of those questions are in the "short" form, and these consist of 50% of the free response score, or 25% of your total score. Long story short, those four questions may not be as ...

  8. Biology Paper 2 Questions and Answers

    Write your name, school, index number in the space provided at the top of the paper. Sign and write the date of examination in the space provided above. This paper consists of two sections, A and B. Answer all the questions in section A in the spaces provided. In section B answer question 6 (compulsory) and either question 7 or 8 in the answer ...

  9. How To Answer Biology Essay Questions

    If you are asked to show your working, do so clearly, making use of the available space to answer the question. Don't skip too many lines of working at once. KCSE 2022 Biology Essay Questions and Answers (KCSE 2022 Prediction Questions) Here are KCSES 2022/2022 Biology Essay Questions and Answers (KCSE 2022 Prediction Questions).

  10. AP Biology Practice Exams

    This is the 2013 practice test published by the College Board. Includes AP Biology multiple choice, grid-ins, and free response questions. Be sure to review this one! This is the 1999 exam that was released by the College Board. Includes answers for multiple choice along with thorough explanations for the essay questions.

  11. Past Papers & Questions by Topic

    We're on a mission to take the guesswork out of Biology revision. Make your way through our clear revision notes, Biology past papers, typical exam questions, fully-explained model answers and more tailored to your Biology specification. Just select your qualification level and exam board below, and dive into everything you'll need to make ...

  12. Biology Paper 3 Questions, Answers and Confidential

    Biology Form 4 Notes; Biology Essays; ... Mock Exam 2022 Questions and Answers; 2023 Mocks; National Schools Past Papers. Alliance Boys High School; Maranda High School; ... Download Biology Paper 3 Questions, Answers and Confidential - KCSE 2022 Mock Exams Set 1. Tap Here to Download for 50/-

  13. How to answer A-level Biology essay questions

    The Synoptic essay questions in paper 3 of the AQA A-level biology course carries 25 marks, so can make a big difference to your overall grade. The essay encourages you to think across different topics (some call this "synoptic" or "holistic" thinking). This is also valuable for other areas of the exams, particularly the application.

  14. Free Biology Essay Questions for Kcse

    Describe process of urine formation in mammalian kidneys. (20mks) Explain the uses of various hormones in homeostasis in mammals. (10mks) Distinguish between diabetes mellitus and insipidus. (7mks) Explain how the various abiotic factors may affect the plant. (10mks) Discuss cause effect and control measures of water pollution. (20mks)

  15. PDF 2022 AP Exam Administration Scoring Guidelines

    Question 1: Interpreting and Evaluating Experimental Results with Experimental Design 9 points. The binding of an extracellular ligand to a G protein-coupled receptor in the plasma membrane of a cell triggers intracellular signaling (Figure 1, A). After ligand binding, GTP replaces the GDP that is bound to Gsα , a subunit of the G protein ...

  16. BIOLOGY Paper 2 Questions and Answers

    State the function of the eustachian tube in the mammalian ear. (1 mark) State the importance of each of the following in the mammalian ear: wax (1 mark) endolymph and perilymph (2 marks. SECTION B (40 marks) Answer question 6 (compulsory) and either question 7 or 8 in the space provided after question 8.

  17. BIOLOGY ESSAY COLLECTION Compiled By: A.L.Miskeen (MSc PGDE SLPS)

    biology famous teacher's model papers (53) biology new syllabus theory (27) biology past papers & schemes (3) biology pilot exams (9) biology question collection (9) biology term exam papers (24) c.maths (53) c.maths famous teacher's model papers (29) c.maths model papers (7) c.maths past papers & schemes (4) c.maths pilot exams (8) c.maths ...

  18. Biology Paper 1 Questions and Answers

    Questions. INSTRUCTIONS. All Questions are Compulsory. Wrong Spelling of Technical Terms shall be Penalized. State TWO ways in which the study of Biology has helped the world in the accelerated fight against the recent Covid-19 pandemic. (2mks. Give ONE function of centrioles (1mk. Name a Kingdom in which all members lack centrioles in their ...

  19. PDF Biology essay titles

    The membranes of different types of cells are involved in many different functions. 2013. 10 b. There are many different types of relationships and interactions between organisms. 2012. 10 a. The importance of shapes fitting together in cells and organisms. 2012. 10 b.

  20. WAEC Biology Questions and Answers 2022/2023 Expo, Theory & OBJ

    Biology Examination Paper Types 2022. WAEC Biology OBJ Paper 1: The WAEC Biology paper one (OBJ) is the objective paper and it comes with 60 multi-choice OBJ questions. Candidates are to are answer these questions within 1 hour 30 minutes. WAEC Biology Theory Paper 2: WAEC Biology paper two is the Biology essay paper which is made up of ...

  21. Biology Essay Questions and Answers

    a) Carbon (IV) Oxide. Required in the dark stage of photosynthesis; it combines with the hydrogen ion from the light stage; to form glucose, proteins and lipids; low concentrations reduce the rate of production of energy and food; while high concentrations leads to an increase in the amount of energy and food formed; b) Light.

  22. 2022 WAEC Biology Objectives and Essay Questions with Solutions

    THEORY QUESTIONS. 1ai)What is a cell? (ii) Name three scientists that are associated with the discovery of the cell. bi) Make a diagram, 6cm to 8cm long of a plant cell and label only the organelle responsible for cell reproduction. ii) Name one blood cell in humans that does not have the organelle in bi).

  23. NECO Biology Questions and Answers 2021/2022 [Theory & OBJ Expo]

    This solution is made available so as to help NECO candidates looking for NECO Biology Questions and Answers 2021 expo solution for paper 1 and paper 2. Download NECO 2021 Timetable PDF Here. I must also let you know that this NECO 2021 Biology Questions and Answers expo solution is free for all NECO candidates. Register NECO Online.